507 Exam 4: Leik Practice Questions

¡Supera tus tareas y exámenes ahora con Quizwiz!

You note that your 11-year-old female patient is at Tanner stage II. You would advise her mother that menarche will probably start in: A) 2 to 3 years B) 3 to 4 years C) 5 years D) It is dependent on the girl's genetic makeup

A) 2 to 3 years Menarche normally begins approximately 2 to 3 years following Tanner stage II (breast budding). Median age of menarche in the United States is 12.43 years. It usually occurs within 2 to 3 years after breast budding (American College of Obstetricians and Gynecologists, 2015).

Which of the following is considered an abnormal finding? A) A "clunk" sound heard while performing the Ortolani maneuver B) A 6-month-old infant who starts to babble C) A 10-year-old boy with aching pain on the front of the thighs that starts in late afternoon or at night D) A 12-month-old who is "cruising"

A) A "clunk" sound heard while performing the Ortolani maneuver The "clunk" sound during the Ortolani maneuver is a positive finding and signifies a possible hip abnormality (hip dysplasia) in infants. Refer the infant to a pediatric orthopedist. Infants start to babble at 6 months. At 12 months, babies learn to "cruise" or to hold onto furniture while walking. Some boys and girls may have "growing pains" on both legs (or both thighs, calves, or behind the knees), which are usually felt in the late afternoon, evening, and/or at night. These symptoms should not interfere with the child's ability to play and affect children from age 3 to 12 years. Stretching exercises of the thighs and hamstrings, massaging the area, and warm packs are helpful. If pain affects only one leg or hip, it is abnormal (rule out bone cancer, sarcoma, leukemia, hip abnormalities). In such cases, refer the patient to a pediatric orthopedist.

All of the following clinical findings are considered benign oral findings except: A) A patch of leukoplakia B) Fordyce spots C) Torus palatinus D) Fishtail uvula

A) A patch of leukoplakia Leukoplakia mainly affects the mucous membranes of the mouth. It is thought to be caused by irritation. Leukoplakia are patches on the tongue, in the mouth, or on the inside of the cheek that occur in response to long-term irritation, including smoking, holding chewing tobacco or snuff in the mouth for a long period, or other tobacco use, especially pipes (smoker's keratosis). Leukoplakia on the tongue is also an early sign of HIV.

A positive obturator sign might signify which of the following conditions? A) Acute appendicitis B) Acute pancreatitis C) Acute cholecystitis D) Acute hepatitis

A) Acute appendicitis A positive obturator sign may indicate acute appendicitis. The test is performed with the patient supine. The examiner rotates the hip, using full range of motion. The test is positive if pain is experienced with movement or flexion of the hip.

Grey-Turner's sign is highly suggestive of which of the following conditions? A) Acute pancreatitis B) Acute appendicitis C) Acute diverticulitis D) Gastric cancer

A) Acute pancreatitis Grey-Turner's sign is the acute onset of bluish discoloration located on the flank area that is caused by bruising. It is usually associated with severe acute pancreatitis, but it can also be found in some cases of ruptured ectopic pregnancy.

A 20-year-old college student reports to the student health clinic with a laceration of her left hand. She tells the nurse practitioner that she cut her hand while working in her garden. Her last Td booster was 5½ years ago. Which of the following is correct? A) Administer a booster dose of the Tdap vaccine B) Administer the Td vaccine and the Td immunoglobulin (HyperTET) C) Administer Td immunoglobulin (HyperTET) only D) She does not need any Td immunoglobulin (HyperTET) or a Td booster

A) Administer a booster dose of the Tdap vaccine The Centers for Disease Control and Prevention (CDC) recommends the Tdap for teens or adults who need a Td booster (once in a lifetime). A tetanus vaccine booster is recommended for recent injuries if it has been more than 5 years since the last dose.

The Pap smear result for a 20-year-old sexually active college student whose partner uses condoms inconsistently reveals a large number of white blood cells and blood along with inflammatory changes. During the speculum exam, the cervix bled very easily (friable), and a small amount of purulent discharge was present on the cervical surface. No cervical motion tenderness was noted during the bimanual vaginal exam. What is the next step in the management of this patient? A) Advise the patient to return to the clinic for chlamydia and gonorrhea testing B) Instruct the patient to use metronidazole vaginal cream C) Tell the patient she needs a repeat Pap smear in 6 months D) Advise the patient to use a Betadine douche at bedtime for 3 days

A) Advise the patient to return to the clinic for chlamydia and gonorrhea testing The patient has signs and symptoms of mucopurulent cervicitis (friable cervix, purulent discharge) and should be tested for chlamydia and gonorrhea. Empiric treatment for mucopurulent cervicitis should be started (doxycycline BID for 7 days) even if these tests are negative, but the patient's cervix appears infected. Douching increases the risk of pelvic inflammatory disease (PID) and is not recommended.

Rovsing's sign is associated with which of the following? A) An acute abdomen, such as during a ruptured appendix B) Knee instability C) Damage to the meniscus of the knee D) Acute cholelithiasis

A) An acute abdomen, such as during a ruptured appendix Rovsing's sign identifies an acute abdomen, such as occurs in acute appendicitis. With the patient in the supine position, the examiner palpates deep into the left lower quadrant of the abdomen. The maneuver is positive if pain is referred to the right lower quadrant.

The Jarisch-Herxheimer reaction is best described as: A) An immune-mediated reaction precipitated by the destruction of a large number of spirochetes due to an antibiotic injection B) Severe chills and elevated blood pressure C) Caused by infection with either Chlamydia trachomatis or gonorrheal infection of the liver capsule D) Associated with certain viral illnesses

A) An immune-mediated reaction precipitated by the destruction of a large number of spirochetes due to an antibiotic injection The dying bacteria release antigens that cause a host reaction. Herxheimer reaction, or Jarisch-Herxheimer reaction, may occur with infections caused by spirochete bacteria such as syphilis and Lyme disease. It is more commonly seen after treatment of early-stage syphilis. It usually occurs in the first 24 hours after therapy. Signs and symptoms are headache, myalgias, rigors, sweating, hypotension, and worsening of rash (if present). No treatment is needed, as it usually resolves within 12 to 24 hours.

A 14-year-old female adolescent is worried that she has not started to menstruate like most of her friends. During the gynecological examination, the nurse practitioner tells the mother, who is in the room with the patient, that her daughter is starting Tanner stage II. What are the physical exam findings during this stage? A) Breast buds and some straight pubic hair B) Fully developed breasts and curly pubic hair C) Breast tissue with the areola on a separate mound with curly pubic hair D) No breast tissue and no pubic hair

A) Breast buds and some straight pubic hair Tanner stage II in females is noted for breast and papilla elevated as a small mound and increased areola diameter (breast buds). Tanner II pubic hair for females is sparse, lightly pigmented straight hair along the medial border of the labia.

According to the Centers for Disease Control and Prevention (CDC), which of the following is the recommended treatment for uncomplicated gonorrheal urethritis infection in an adult man? A) Ceftriaxone (Rocephin) 250 mg IM plus azithromycin 1 g orally B) Valacyclovir (Valtrex) 500 mg PO BID × 10 days C) Ceftriaxone (Rocephin) 250 mg IM plus doxycycline 100 mg orally BID for 14 days D) One dose of oral fluconazole (Diflucan) 150 mg

A) Ceftriaxone (Rocephin) 250 mg IM plus azithromycin 1 g orally Drug therapy is based on the 2015 Centers for Disease Control and Prevention (CDC; 2015) sexually transmitted disease guidelines. In patients with gonorrheal infection, always co-treat for chlamydia.

Which of the following is a true statement regarding acute gastritis? A) Chronic intake of nonsteroidal anti-inflammatory drugs (NSAIDs) can cause the disorder B) Chronic lack of dietary fiber is the main cause of the disorder C) The screening test for the disorder is the barium swallow test D) The gold standard to evaluate the disorder is a colonoscopy

A) Chronic intake of nonsteroidal anti-inflammatory drugs (NSAIDs) can cause the disorder Signs and symptoms of gastritis are nausea/vomiting, upset stomach, loss of appetite, and burning/aching or gnawing pain located in the epigastric area. Nonselective NSAIDs (aspirin, ibuprofen, naproxen, others) have adverse effects on the gastrointestinal (GI) tract, kidneys, central nervous system, and cardiovascular effects, and decrease platelet aggregation (aspirin). Chronic use of nonselective NSAIDs disrupts the production of prostaglandins, which involves cycloxygenase-1 (COX-1) and COX-2. The GI mucosa uses COX-1 to produce mucosal protective factors. Blocking COX-1 decreases these protective factors and increases risk of gastritis, ulcers, and GI bleeding. Selective NSAIDs, such as celecoxib (Celebrex), do less damage to the GI tract because they block only COX-2, which is responsible for pain and inflammation.

Which of the following is a sensitive screening test for human immunodeficiency virus (HIV)? A) Combination HIV-1 and HIV-2 antibody immunoassay with p24 antigen B) Western blot test C) ELISA test D) HIV antibody test

A) Combination HIV-1 and HIV-2 antibody immunoassay with p24 antigen The Centers for Disease Control and Prevention (CDC) recommend screening with the combination antigen/antibody immunoassay with p24 antigen test. Previously, the ELISA (enzyme-linked immunosorbent assay) was used as the screening test, and the Western blot was the confirmatory test. The ELISA and Western blot tested only for the HIV antibody. The CDC recommends testing everyone between the ages of 13 and 64 years for HIV at least once as part of routine health care. If risk factors are present, the patient should be tested for HIV annually. For sexually active gay and bisexual men, the CDC recommends more frequent testing, perhaps every 3 to 6 months (CDC, 2016).

All of the following findings are associated with the secondary stage of an infection by the organism Treponema pallidum except: A) Condyloma acuminata B) Maculopapular rash of the palms and soles C) Lymphadenopathy D) Condyloma lata

A) Condyloma acuminata Findings consistent with the diagnosis of syphilis, caused by the Treponema pallidum organism, include painless chancre, maculopapular rash of the palms and soles, lymphadenopathy, and condyloma lata. Condyloma acuminata (genital warts) are caused by the human papillomavirus (HPV) and spread to others by skin-to-skin contact.

All the following signs and symptoms are associated with irritable bowel syndrome except: A) Diarrhea with blood mixed in the stool B) Diarrhea and/or constipation C) Abdominal pain relief after defecation D) Mucus with stools

A) Diarrhea with blood mixed in the stool Irritable bowel syndrome (IBS) is considered a functional disorder because the colon tissue is normal. There is no inflammation or bleeding in IBS. Patients report chronic abdominal pain, flatulence, bloating, and changes in bowel movements. IBS is more common in women than men. Rectal bleeding, blood in stool, anemia, fatigue, and weight loss are associated with inflammatory bowel disease (IBD), such as Crohn's disease or ulcerative colitis.

Which of the following is recommended treatment for erythema migrans or early Lyme disease? A) Doxycycline (Vibramycin) 100 mg PO BID × 21 days B) Ciprofloxacin (Cipro) 250 mg PO BID × 14 days C) Erythromycin (E-mycin) 333 mg PO TID × 10 days D) Dicloxacillin (Dynapen) 500 mg PO BID × 10 days

A) Doxycycline (Vibramycin) 100 mg PO BID × 21 days Erythema migrans is the rash characteristic of Lyme disease and it usually appears 7 to 10 days after a tick bite. Lyme disease is caused by Borrelia burgdorferi, a spirochete. The rash appears either as a single expanding red patch or a central spot surrounded by clear skin that is in turn ringed by an expanded red rash (bull's eye). The choice of antibiotic depends on bacterial sensitivity. Doxycycline 100 mg BID for 14 to 21 days is the recommended treatment of adults.

All of the following measures have been found to help lower the risk of osteoporosis except: A) Drinking organic orange juice B) Eating low-fat dairy foods C) Performing weight-bearing exercises D) Vitamin D supplementation

A) Drinking organic orange juice Commercial orange juice is fortified with calcium and vitamin D. But organic products usually do not have any additives such as calcium. Vitamin D levels must be sufficient for the body to absorb calcium. Eating foods high in vitamin D and calcium along with calcium and vitamin D supplements is advised to protect the bones and prevent bone loss. Performing weight-bearing exercises daily also increases bone strength.

The mother of an 8-year-old boy reports the presence of a round red rash on the child's left lower leg. It appeared 1 week after the child returned from visiting his grandparents, who live in Massachusetts. During the skin exam, the maculopapular rash is noted to have areas of central clearing making it resemble a round target. Which of the following is best described? A) Erythema migrans B) Rocky Mountain spotted fever C) Meningococcemia D) Larva migrans

A) Erythema migrans Erythema migrans is a symptom of early Lyme disease. It is an annular lesion that slowly enlarges with time (days to weeks) and has central clearing. It is caused by a bite from an infected (Borrelia burgdorferi) blacklegged tick. If untreated, infection will spread to joints, nervous system, and heart. Most cases of Lyme disease occur in the Northeast, mid-Atlantic states, Wisconsin, Minnesota, and northern California.

All of the following statements are correct regarding the Td vaccine except: A) Fever occurs in up to 80% of the patients B) A possible side effect is induration on the injection site C) The Td is given every 10 years D) The DPT and DT should not be given beyond the seventh birthday

A) Fever occurs in up to 80% of the patients Side effects of the Td vaccine include induration at the injection site. Td is given in adults every 10 years. The DPT and DT should not be given beyond 7 years of age. Fever may occur, but studies do not support 80% of patients having fever.

What type of infection is caused by the human parvovirus B19? A) Fifth disease B) Roseola infantum C) Condyloma acuminata D) Scarlet fever

A) Fifth disease Fifth disease (erythema infectiosum) is caused by infection with the human parvovirus. It is spread by aerosolized droplets (cough, sneeze). The most commonly affected age group is children and adolescents between the ages of 5 and 15 years. Symptoms may take 1 to 3 weeks to resolve. There are three stages: The first stage is characterized by a bright-red rash on the cheeks/face with slight fever and coldlike symptoms (coryza); the second stage by a generalized rash that appears lacelike; and in the third stage the rash disappears but reoccurs when the child is exposed to sunlight, heat, or exercise. Treatment is symptomatic.

Which of the following conditions is associated with three stages of rashes? A) Fifth disease B) Erythema infectiosum C) Varicella D) Rocky Mountain spotted fever

A) Fifth disease Fifth disease has three stages. The prodromal stage begins with symptoms of an upper respiratory infection, such as low-grade fever, headache, chills, and malaise. In the second stage, a red rash appears on the cheeks, known as the "slapped cheek" rash. This usually resolves in 2 to 3 days. In the third stage, the rash moves to the arms and legs and becomes a lacy-appearing rash that is flat and appears purple; this may last for a few weeks.

An 18-year-old male who recently returned from a camping vacation is complaining of abrupt onset of fever, abdominal cramps, watery diarrhea, foul flatus, vomiting, and anorexia for 2 days. He has explosive diarrhea with a very strong "foul" odor. He reports that he remembers swallowing lake water a few times while swimming. Which of the following conditions is most likely? A) Giardiasis B) Irritable bowel syndrome C) Crohn's disease D) Celiac disease

A) Giardiasis Giardia lamblia is a protozoan with flagella that infects the small intestine. Giardiasis is common in the developing world and rural areas and can cause acute and chronic infection. Campers, hunters, backpackers, and travelers to endemic areas are at highest risk. The incubation period is 7 to 14 days, and symptoms can last for 2 to 4 weeks. Chronic infection can result in malabsorption and significant weight loss. First-line antibiotics are tinidazole (Tindamax) or nitazoxanide (Nizonide). Alternative antibiotics are metronidazole (Flagyl) and others.

Women with polycystic ovary syndrome (PCOS) are at higher risk for the following: A) Heart disease and endometrial cancer B) Uterine fibroids and ovarian cancer C) Premature menopause D) Pelvic inflammatory disease (PID)

A) Heart disease and endometrial cancer Chronic anovulation results in high levels of estrogen and androgens in the body. The risk of heart attack is four to seven times higher in women with polycystic ovary syndrome (PCOS) than women of the same age without PCOS. Women with PCOS are at greater risk of having high blood pressure and have high levels of low-density lipoprotein (LDL) and low levels of highdensity lipoprotein (HDL) cholesterol. Lack of ovulation is usually the reason for fertility problems in women with PCOS. More than 50% of women with PCOS have diabetes or prediabetes (impaired glucose tolerance) before the age of 40. Women with PCOS are also at risk for endometrial cancer. Irregular menstrual periods and the lack of ovulation cause women to produce the hormone estrogen, but not progesterone. Without progesterone, the endometrium becomes thick, which can cause heavy or irregular bleeding. Over time, this can lead to endometrial hyperplasia and cancer.

What is the name of the immune process that is responsible for anaphylactic reactions? A) IgE-mediated reaction B) IgG-mediated reaction C) Antibody reaction D) Atopic reaction

A) IgE-mediated reaction Anaphylaxis is an immunoglobulin E (IgE)-mediated reaction (also known as IgE-mediated type 2). IgE immediate reactions, such as anaphylaxis, trigger mast cell degranulation and release of potent mediators, such as histamines, leukotrienes, and prostaglandins, that immediately induce constriction of smooth muscle, swelling, vasodilation, and other pathological changes in the body that may be fatal.

All of the following vaccines are contraindicated in pregnant women except: A) Influenza B) Mumps C) Varicella D) Rubella

A) Influenza Not all vaccinations are safe to get during pregnancy. However, the flu vaccine can be given before, during, and after pregnancy. The other vaccines are live viruses and are contraindicated during pregnancy.

A 20-year-old Asian man reports pain in his right knee after twisting it while playing soccer. The injured knee locks up when he attempts to straighten his leg. Which of the following conditions is most likely? A) Injury to the meniscus of the right knee B) Injury to the patella of the right knee C) Injury to the ligaments of the right knee D) Rupture of the quadriceps tendon

A) Injury to the meniscus of the right knee Pain in the knee with "locking up" while attempting to straighten the leg is highly suggestive of injury to the meniscus. The best imaging test for meniscus injury is the MRI.

What is the significance of a positive Lachman sign? A) Instability of the knee caused by damage to the anterior cruciate ligament of the knee B) Posterior cruciate ligament laxity which may cause locking of the knee C) Achilles tendon rupture D) Patellar tendon rupture

A) Instability of the knee caused by damage to the anterior cruciate ligament of the knee A positive Lachman sign is highly suggestive of damage to (i.e., rupture of) the anterior cruciate ligament (ACL) of the knee. The anterior drawer sign may also be positive. The examiner notes laxity of the abnormal knee joint compared with the normal knee. The Lachman test or maneuver is considered more sensitive for ACL damage than the anterior drawer test.

When Molluscum contagiosum is found on the genital area of children, which of the following is the best explanation? A) It should raise the suspicion of child sexual abuse B) It is not considered a sexually transmitted disease C) It is caused by atypical bacteria D) It is caused by the poxvirus and will resolve on its own

A) It should raise the suspicion of child sexual abuse Molluscum contagiosum is spread by skin-to-skin contact. Lesions found in the genital area of young children should be evaluated for suspicion of child sexual abuse.

A red, raised serpiginous-shaped rash is noted by the nurse practitioner on the right foot of a 4-year-old child brought in for a preschool physical by the mother. The child complains of severe itch and keeps scratching the lesion. The mother reports that the child frequently plays in the yard without wearing shoes or sandals. Which of the following is most likely? A) Larva migrans B) Erythema migrans C) Tinea pedis D) Insect bites

A) Larva migrans Larva migrans results from infection with the eggs of parasites (worms) that are commonly found in the intestines of dogs and cats. Children are at high risk of developing this infection if they come in contact with dirt that is contaminated with dog or cat feces. Eating foods that are grown in contaminated soil and/or raw liver are other means of transferring the infection. After the eggs hatch, the parasite can migrate to other organs of the body if left untreated.

Human papillomavirus (HPV) infection of the larynx has been associated with: A) Laryngeal cancer B) Esophageal stricture C) Cervical cancer D) Metaplasia of the squamous cells

A) Laryngeal cancer Human papillomavirus (HPV) exposure is a risk factor in laryngeal cancer. HPV DNA transforms the moist membranes of epithelial cells (cervix, anus, mouth, and throat). The juvenile type is related to vertical transmission and the adult type to orogenital contact. HPV subtype 16 accounts for the majority of oral tumors, oropharynx cancers, and laryngeal cases.

Which of the following is considered by Latinos/Hispanics to be a spiritual illness that can cause symptoms such as loss of appetite, crying, diarrhea, and weakness or death among infants and small children? A) Mal ojo or mal de ojo B) Chronic nightmares C) Trabajo D) Malo

A) Mal ojo or mal de ojo Mal ojo or mal de ojo is a spiritual illness that can cause symptoms such as loss of appetite, crying, diarrhea, colic, fear, weakness, or death. A curandero or curandera is usually consulted and does spiritual cleansing of the patient. It may take several cleansings (limpia) to cure the patient. Trabajo means "work" and malo means "bad" in Spanish; these are included as distractors.

A 22-year-old woman who is a member of the track team is seen by the nurse practitioner in the college health clinic with a complaint of "shin splints." She states that she has recurrent localized pain on the anterior aspect of the left shin that is aggravated by running and jumping. She recently increased the intensity of her training schedule because she is planning to participate in a local marathon. During the physical examination, the nurse notes a focal area of tenderness on the anterior medial aspect of the left tibia. Which of the following conditions is most likely? A) Medial tibial stress fracture B) Morton's neuroma C) Acute tendinitis D) Osgood-Schlatter disease

A) Medial tibial stress fracture Medial tibial stress fracture and medial tibial stress syndrome (MDSS, or "shin splints") are more common in individuals who participate in sports that involve running and/or jumping (track and field, soccer, basketball). Medial tibial stress fracture is more common in female athletes. It is caused or exacerbated by excessive training, sudden increase in training intensity, or overuse. The symptoms of medial tibial stress fracture can resemble MDSS, but the pain is more persistent and worsens until it also occurs at rest. The physical exam findings reveal a focal area of tenderness on the anterior medial aspect of the tibia. The imaging test of choice is MRI. A radiograph (x-ray) will not show a stress fracture. If a medial tibial stress fracture is suspected, the patient should be advised to stop the sport/activity, use RICE (rest, ice, compression, elevation) to relieve symptoms, and should be referred to an orthopedic specialist.

A 19-year-old male athlete complains of acute knee pain after a football game. The nurse practitioner elicits McMurray's sign, which is positive on the patient's injured knee. This is a test for: A) Meniscal injury B) Inflammation of the knee joint C) Osteophytes of the knee joint D) Tenosynovitis

A) Meniscal injury With an acute knee injury, the knee should be assessed using McMurray's sign. A positive McMurray's sign indicates a meniscal injury. Inflammation of the knee, osteophytes, and tenosynovitis would not elicit a positive McMurray's sign.

A 33-year-old man is diagnosed with a mild case of Clostridium difficile infection of the colon. The patient reports a history of hospitalization the previous month. Which of the following regimens is the preferred treatment? A) Metronidazole (Flagyl) 500 mg orally TID × 10 to 14 days B) Azithromycin 250 mg orally once daily × 5 days (Z-Pack) C) Vancomycin 125 mg orally QID × 10 to 14 days D) Treatment with antibiotics is not recommended

A) Metronidazole (Flagyl) 500 mg orally TID × 10 to 14 days The first-line antibiotic for patients with mild to moderate Clostridium difficile colitis is metronidazole. Vancomycin is reserved for patients with severe cases.

Orchitis is caused by which of the following? A) Mumps virus B) Measles virus C) Chlamydia trachomatis D) Chronic urinary tract infections that are not treated adequately

A) Mumps virus Orchitis (infection of the testicle) is caused by the mumps virus. It can cause sterility in males.

A 30-year-old chef complains of pruritic hives over her chest and arms, but denies difficulty swallowing or breathing. She reports a family history of allergic rhinitis and asthma. Which of the following interventions is most appropriate? A) Obtain a complete and thorough history B) Recommend an oral antihistamine such as diphenhydramine 25 mg PO QID C) Give an injection of epinephrine 1:1000 intramuscularly stat D) Call 911

A) Obtain a complete and thorough history Before prescribing medications, a thorough history must be obtained to determine possible causes of hives. The patient denied difficulty with swallowing and breathing, so there was no medical emergency that would require calling 911.

A 25-year-old woman complains of dysuria, severe vaginal pruritus, and a malodorous vaginal discharge. Pelvic examination reveals a strawberry-colored cervix and frothy yellow discharge. Microscopic examination of the discharge reveals mobile organisms that have flagella. The correct pharmacological therapy for the condition is: A) Oral metronidazole (Flagyl) B) Ceftriaxone sodium (Rocephin) injection C) Doxycycline hyclate (Vibramycin) D) Clotrimazole (Gyne-Lotrimin) cream or suppositories

A) Oral metronidazole (Flagyl) Trichomoniasis symptoms include dysuria, severe vaginal pruritus, and malodorous vaginal discharge. Wet prep microscopic examination should show trichomonads that are pear-shaped and have several flagella (whiplike tails) at one end. The Centers for Disease Control and Prevention (CDC) recommendation for treatment is metronidazole. A single dose is effective treatment in most cases of Trichomonas infections.

A teenage girl complains to you of a 1-week episode of dysuria, frequency, and a strong odor to her urine. This is her second episode of the year. The previous urinary tract infection occurred 3 months ago. What is the most appropriate follow-up for this patient? A) Order a urinalysis and urine for culture and sensitivity (C&S), and treat the patient with antibiotics B) Order a urine C&S and hold treatment until you get the results from the lab C) Treat the patient with a 7-day course of antibiotics and order a urine C&S now and after she completes her antibiotics D) Treat the patient with a stronger drug, such as ofloxacin (Floxin), for 10 days

A) Order a urinalysis and urine for culture and sensitivity (C&S), and treat the patient with antibiotics Because this is the second urinary tract infection for the year and the last episode was 3 months ago, the best action is to order the urinalysis and urine C&S to identify the organism causing the infection. Antimicrobial-resistant strains are increasing. Start empiric treatment with an antibiotic for 7-day duration (do not use 3-day regimen).

A middle-aged woman who works in the housekeeping department of a hospital presents to the employee health clinic with a complaint of a needlestick to her left thumb. The needle was in one of the garbage bags from the emergency department. The patient had a little bleeding that stopped spontaneously. Which of the following is the next step? A) Order an enzyme-linked immunosorbent assay (ELISA) test as soon as possible B) Recommend a tetanus booster in 1 week C) Offer the patient hepatitis B immunoglobulin D) Order a chest x-ray

A) Order an enzyme-linked immunosorbent assay (ELISA) test as soon as possible Employee health clinic protocols for needlesticks recommend ordering an ELISA test as soon as possible to establish baseline blood work for the employee.

All of the following are included in the criteria used to diagnose patients with AIDS except: A) Profound fatigue B) Thrush C) Kaposi's sarcoma D) Hairy leukoplakia of the tongue

A) Profound fatigue Criteria used to diagnose AIDS include hairy leukoplakia of the tongue, Kaposi's sarcoma, and thrush.

A 25-year-old man who was involved in a car accident is brought to the local emergency department. He reports wearing a seat belt and was the driver of the vehicle. The patient is complaining of pain on his right lower leg. The right foot is abducted and the ankle is swollen and bruised. There is a small laceration on the ankle. The patient complains of severe right ankle pain when standing. Which of the following tests would be the initial choice for evaluating for possible fractures on the right lower leg? A) Plain radiographs of the right lower leg, ankle, and foot B) Ultrasound of the right lower leg C) MRI of the right ankle D) Radiograph of the right ankle and knee with special view of the hip

A) Plain radiographs of the right lower leg, ankle, and foot Plain radiographs or x-rays are the first exam ordered for suspected or obvious bony fractures. Severe pain on weight bearing and bruising of the right ankle is highly suggestive of a fracture. MRI scan is not an initial test because it is very expensive. MRIs are preferred for joints and soft tissue.

A 55-year-old woman who has had type 2 diabetes for 20 years is concerned about her kidneys. She has a history of three urinary tract infections (UTIs) within the past 8 months, but is currently asymptomatic. Which of the following is the best course to follow? A) Recheck urine during the visit, send a urine specimen for culture and sensitivity, and refer to a nephrologist B) Order a urinalysis dipstick test to be repeated monthly C) Order a CT scan of the kidneys D) Provide empiric treatment for a UTI

A) Recheck urine during the visit, send a urine specimen for culture and sensitivity, and refer to a nephrologist Although the patient is currently asymptomatic, her history of three urinary tract infections (UTIs) in 8 months warrants testing while she is in the office. A UTI is defined as the presence of 100,000 organisms per milliliter of urine in asymptomatic patients or greater than 100 organisms per milliliter of urine with pyuria (>7 WBCs/mL) in a symptomatic patient. Diabetic patients are at higher risk for UTIs and over time may develop bladder damage (cystopathy) and nephropathy. A nephrology consult is prudent.

A 20-year-old Asian man reports pain in his right knee after twisting it while playing soccer. The injured knee locks up when he attempts to straighten his leg. Which of the following actions is the best course for this patient? A) Refer him to an orthopedic specialist B) Refer him to a chiropractor C) Advise him that the clicking noise will resolve within 2 to 4 weeks D) Advise him to use an elastic bandage wrap during the first 2 weeks for knee support and to see you again for reevaluation

A) Refer him to an orthopedic specialist Referral to orthopedics is advised for evaluation of the need for treatment and surgery (arthroscopic repair). If the tear is minor and the pain and other symptoms resolve quickly, muscle-strengthening exercises may be all that is needed to recover fully. In this case, a patient is usually referred to physical therapy. A large meniscus tear that causes symptoms or mechanical problems with the function of the knee joint may require arthroscopic surgery for repair.

A 28-year-old male nurse tells the employee health nurse practitioner that he was treated for a urinary tract infection twice the previous year. The patient denies fever, flank pain, or urethral discharge during the visit. Which of the following is the best follow-up for this patient? A) Refer the patient to a urologist B) Prescribe the patient ofloxacin (Floxin) for 2 weeks instead of 1 week C) Advise the patient that he needs to void every 2 hours when awake D) Refer the patient to the local emergency department, because he has a very high risk of sepsis

A) Refer the patient to a urologist Urinary tract infections (UTIs) are relatively rare in young men with normal urinary tracts. Urethritis can mimic a UTI; rule out gonorrhea and chlamydia. A urine culture before initiating antibiotics and a test of cure is recommended for this group. Therefore, they should be referred to a urologist for evaluation and treatment. Additional tests that should be conducted may include intravenous pyelography, renal ultrasound, CT scan, or cystoscopy.

The nurse practitioner examines a 4-week-old boy whose mother reports that he has cried for at least 3 hours a day at the same time of day since birth. What is the main goal in the clinical evaluation of this infant? A) Rule out any physiological cause for the crying spells B) Make sure that the infant is well clothed C) Evaluate the environment D) Order laboratory and diagnostic testing

A) Rule out any physiological cause for the crying spells For infants who cry for several hours during the day, ruling out a physiological problem that may be causing the distress is recommended.

Which of the following would be classified as a second-degree burn? A) Severe sunburn with blistering B) Burns that involve the subcutaneous layer of skin C) Reddened finger after touching a hot iron D) Burns that involve eschar

A) Severe sunburn with blistering Burns are described according to the depth of injury to the dermis: First degree—erythema without blistering, painful; second degree—red skin with superficial blisters, painful; third degree—entire skin layering, involving the subcutaneous tissue and may include soft tissue facia, painless.

Erysipelas is an infection of the skin most commonly caused by which of the following class of organisms? A) Streptococci B) Staphylococci C) Gram-negative bacteria D) Fungi

A) Streptococci Erysipelas is a skin infection commonly caused by group A beta hemolytic streptococci. This infection is usually more superficial than other bacterial infections of the skin, such as cellulitis.

A 14-year-old girl who is sexually active is brought to the health clinic by her mother for an immunization update. According to the mother, her daughter has had one dose of hepatitis B vaccine. Which of the following vaccines would you administer at this visit? A) Tdap, hepatitis B, and HPV vaccine B) DTaP (diphtheria, tetanus, acellular pertussis) and hepatitis B C) Hepatitis B only D) MMR (measles, mumps, rubella), Td, and HPV vaccine

A) Tdap, hepatitis B, and HPV vaccine There are two types of HPV vaccine. Gardasil can be used for both girls and boys, but Cervarix can only be used for females. HPV vaccine is recommended for preteen boys and girls at age 11 or 12 years. Young women can get HPV vaccine until age 27 years and men can get the HPV vaccine until age 22 years.

Which of the following clinical findings can mimic a case of testicular torsion but is not considered an emergent condition? A) The "blue dot" sign B) One swollen testicle with yellow-colored penile discharge C) Acute onset of dysuria and frequency D) A varicocele

A) The "blue dot" sign The blue dot sign describes a tender nodule located underneath the skin of the testicle that appears as a round, blue-to-purple mass. Also known as torsion of testicular appendage, it is not an emergent condition. It is the most common cause of testicular area pain in younger boys. Treatment is symptomatic.

A 20-year-old White man is being seen for a physical exam by the nurse practitioner. He complains of pruritic macerated areas in his groin that have been present for the past 2 weeks. Which of the following is the most likely? A) Tinea cruris B) Tinea corporis C) Tinea capitis D) Tinea pedis

A) Tinea cruris Tinea cruris (jock itch) is a common skin infection that is caused by a type of fungus called tinea. The fungus thrives in warm, moist areas of the body and, as a result, infection can affect the genitals, inner thighs, and buttocks. Infections occur more frequently in the summer or in warm, wet climates. Tinea cruris appears as a red, itchy rash that is often ring shaped. Tinea corporis involves the body, tinea capitis involves the head, and tinea pedis involves the feet.

Fetal TORCH infections can cause microcephaly, mental retardation, hepatosplenomegaly, and intrauterine growth retardation. The acronym TORCH stands for: A) Toxoplasma gondii, other infections, rubella, cytomegalovirus, and herpes B) Toxic shock syndrome, ocular infections, rubella, cytomegalovirus, and herpes zoster C) Tetanus, ophthalmic infections, roseola, cancer, and head abnormalities D) Toxins, other infections, roseola, candidiasis, and head abnormalities

A) Toxoplasma gondii, other infections, rubella, cytomegalovirus, and herpes The acronym TORCH stands for Toxoplasma gondii, other infections, rubella, cytomegalovirus, and herpes. Although several of the conditions listed in the other answer options can also cause fetal problems, they do not comprise the TORCH disorders.

An 8-year-old boy with type 1 diabetes is being seen for a 3-day history of urinary frequency and nocturia. He denies flank pain and is afebrile. The urinalysis result is negative for blood and nitrites but is positive for a large amount of leukocytes and ketones. He has a trace amount of protein. Which of the following is the best test to order initially? A) Urine for culture and sensitivity B) 24-hour urine for protein and creatinine clearance C) 24-hour urine for microalbumin D) Intravenous pyelogram

A) Urine for culture and sensitivity An 8-year-old male patient with the diagnosis of diabetes has a high risk of urinary tract infections (UTIs). A large amount of leukocytes in the urinalysis is abnormal, and he has been having symptoms of frequency and nocturia for the past 3 days. The urine culture would be ordered because he has a high risk of infection. The urine culture and sensitivity (C&S) is the best evaluation for diagnosing a UTI.

A new mother is planning to breastfeed her newborn infant for at least 6 months. She wants to know whether she should give the infant vitamins. Which of the following vitamin supplements is recommended by the American Academy of Pediatrics (APA) during the first few days of life? A) Vitamin D drops B) Vitamin E drops C) It is not necessary to give breastfed infants vitamin supplements because breast milk contains enough vitamins and minerals that are necessary for the infant's growth and development D) Folic acid drops

A) Vitamin D drops According to the American Academy of Pediatrics (APA), all infants should be given vitamin D supplementation within the first few days of life. Mothers who plan to breastfeed their infants should be taught how to use vitamin D drops. Infant formula is supplemented with vitamin D (and many other vitamins, minerals, and omega-3 oil), so there is no need to give it separately.

Which of the following is the best method for diagnosing a Candida albicans infection of the vagina in the primary care setting? A) Wet smear B) Tzanck smear C) KOH (potassium hydroxide) smear D) Clinical findings only

A) Wet smear The wet smear or wet prep is best used to diagnose Candida albicans infection of the vagina (candidiasis) in the primary care setting. To perform the wet smear or wet prep, a small amount of vaginal discharge is obtained and smeared on a slide, then one to two drops of normal saline solution is applied. Candida albicans is a type of yeast organism (fungi). Candida viewed under the microscope will have pseudohyphae and spores. The specimen may also include white blood cells because Candida causes an inflammatory reaction in the vagina.

Which of the following laboratory tests is positive in a large number of patients with systemic lupus erythematosus? A) Antinuclear antibody (ANA) B) Rheumatoid factor C) Antiparietal antibody D) Immunoglobulin

Antinuclear antibody (ANA) Screening tests for systemic lupus erythematosus include ANA. The rheumatoid factor test is performed to diagnose rheumatoid arthritis. Antiparietal antibody testing is done to evaluate for antibodies against the parietal cells. The parietal cells make a substance that the body needs to absorb vitamin B. Immunoglobulin testing is done to assess for the amount of antibodies in the blood for a specific disease.

All of the following children are within the parameters of normal growth and development for their age group except: A) A 2-month-old who coos and smiles B) A 14-month-old who understands complex commands C) A 20-month-old who can walk without support D) A 3-year-old who can speak in three- to four-word sentences

B) A 14-month-old who understands complex commands A 14-month-old child should developmentally be able to say "mama" and "dada," know his own name, and know at least two to four words. A 2-year-old is able to understand simple commands.

A bulla is defined as: A) A solid nodule less than 1 cm in size B) A superficial vesicle filled with serous fluid greater than 1 cm in size C) A maculopapular lesion D) A shallow ulcer

B) A superficial vesicle filled with serous fluid greater than 1 cm in size This is a blister—a circumscribed, fluid-containing, elevated lesion of the skin, usually more than 5 mm in diameter.

A positive psoas and obturator sign is highly suggestive of which of the following conditions? A) Ectopic pregnancy B) Acute appendicitis C) Peritonitis D) Abdominal aortic aneurysm

B) Acute appendicitis Both the psoas and obturator signs are associated with acute appendicitis. When the appendix becomes inflamed or ruptured, the blood and pus irritate the psoas and/or obturator muscles, which are both located in the retroperitoneal area. Both muscles are hip flexors and assist with hip movement.

Extreme tenderness and involuntary guarding at McBurney's point is a significant finding for possible: A) Acute cholecystitis B) Acute appendicitis C) Acute gastroenteritis D) Acute diverticulitis

B) Acute appendicitis Symptoms of an acute abdomen, such as appendicitis, include extreme tenderness and involuntary guarding at McBurney's point. McBurney's point is the name given to the point over the right side of the abdomen that is one third of the distance (approximately 2 inches) from the anterior superior iliac spine to the umbilicus. This point roughly corresponds to the most common location of the base of the appendix where it is attached to the cecum. (During pregnancy the location of the appendix changes as the uterus grows.)

The nurse practitioner would test the obturator and iliopsoas muscle to evaluate for: A) Cholecystitis B) Acute appendicitis C) Inguinal hernia D) Gastric ulcer

B) Acute appendicitis Signs and symptoms of an acute abdomen include involuntary guarding, rebound tenderness, boardlike abdomen, and a positive obturator and psoas sign. A positive obturator sign occurs when pain is elicted by internal rotation of the right hip from 90 degrees hip/knee flexion. The psoas sign is positive when pain occurs with passive extension of the thigh while the patient is lying on his or her side with knees extended, or when pain occurs with active flexion of the thigh at the hip

A concerned new mother reports to you that her son, who is 3 years of age, is not toilet trained yet. Which of the following is an appropriate reply? A) Recommend a referral to a pediatric urologist B) Advise the mother that her child is developing normally C) Recommend a bed-wetting alarm D) Recommend a voiding cystogram

B) Advise the mother that her child is developing normally Toilet-training begins at approximately 2 years of age and may take 1 to 2 years to complete. Boys who are not toilet-trained by 3 years of age may still be developing normally.

A mother brings in her 6-year-old daughter to see the nurse practitioner (NP). She complains that the school nurse found a few nits in her daughter's hair. The mother states that the school has a "no nits" policy regarding head lice and her daughter cannot go back to school until all the nits have been removed. The child was treated with permethrin shampoo (Nix) twice about 3 months ago. During the physical exam, the NP sees a few nits that are about 2 inches away from the scalp. The child denies itchiness on her scalp. Which of the following is the best action for the NP to follow? A) Prescribe lindane (Kwell) for the child because she may have head lice that are resistant to permethrin B) Advise the mother to use a nit comb after spraying the child's hair with white distilled vinegar, wait for 15 minutes, and then rinse the hair C) Advise the mother to retreat the child with permethrin cream instead of shampoo D) Reassure the mother that the nits will probably drop off after a few weeks

B) Advise the mother to use a nit comb after spraying the child's hair with white distilled vinegar, wait for 15 minutes, and then rinse the hair According to the Centers for Disease Control and Prevention (CDC), nits that are more than ¼ inch from the scalp are usually not viable. The child also does not have an itchy scalp. One method of removal is to soak the patient's head with distilled vinegar (and then rinse after), which will break down the protein of the nit casings, making it easy to comb them out of the hair.

Which of the following factors is associated with increased risk of osteopenia in teenage girls? A) Drinking one glass of low-fat milk daily B) Anorexia nervosa C) Participation in sports D) A normal BMI (body mass index)

B) Anorexia nervosa Anorexia nervosa increases the risk for osteopenia in teenage girls as a result of the poor intake of foods high in calcium and vitamin D.

Which of the following is the treatment for nongonococcal urethritis? A) Ceftriaxone (Rocephin) 250 mg IM B) Azithromycin 1 g PO in a single dose C) Doxycycline 100 mg PO BID × 14 days D) Treatment is not necessary

B) Azithromycin 1 g PO in a single dose There are two regimens for treatment of nongonococcal urethritis: either azithromycin 1 g PO in a single dose or doxycycline 100 mg PO BID for 7 days. Before treatment, test the patient for chlamydia and gonorrhea. Use the first urine of the day to test the patient with a nucleic acid amplification test (NAAT). The bacteria that may cause this condition are Chlamydia trachomatis, Ureaplasma urealyticum, Mycoplasma genitalium, and Trichomonas vaginalis. There is no test for the last three pathogens mentioned.

A 30-year-old woman who is sexually active complains of a large amount of milk-like vaginal discharge for several weeks. A microscopy slide reveals a large number of squamous epithelial cells that have blurred margins. Very few white blood cells are seen. The vaginal pH is at 6.0. What is most likely? A) Trichomonas infection B) Bacterial vaginosis C) Candidal infection D) A normal finding

B) Bacterial vaginosis In bacterial vaginosis (BV), the normal hydrogen peroxideproducing lactobacilli are replaced by an overgrowth of anaerobic bacteria. BV is a vaginosis (does not cause inflammation) rather than a vaginitis. No symptoms may be present or the patient's history may include strong vaginal odor ("fishy" odor) and increased vaginal discharge (milky white, thin, adherent discharge or dull gray discharge). Findings include alkaline pH greater than 4.5, the presence of clue cells (wet smear using microscopy), and positive "whiff test" (a strong "fishy" odor when vaginal discharge is mixed with one drop of potassium hydroxide [KOH]). Clue cells are squamous epithelial cells dotted with large numbers of bacteria that obscure borders.

Fitz-Hugh-Curtis syndrome is associated with which following infection? A) Syphilis B) Chlamydia trachomatis C) Herpes genitalis D) Lymphogranuloma venereum

B) Chlamydia trachomatis Fitz-Hugh-Curtis syndrome (or perihepatitis) is a complication of pelvic inflammatory disease (PID). It is more common with chlamydial PID, but it can also occur with gonorrheal PID infection. It is caused by inflammation of the liver capsule, which leads to scarring. Signs and symptoms include sharp pain on the right upper quadrant (pleuritic-like) with PID symptoms. Like PID, it is treated with antibiotics.

Which of the following is the most common cause of nongonococcal urethritis? A) Escherichia coli B) Chlamydia trachomatis C) Neisseria gonorrhoeae D) Mycoplasma genitalium

B) Chlamydia trachomatis The most common cause of nongonococcal urethritis is Chlamydia trachomatis, but it can also be caused by Ureaplasma urealyticum, Haemophilus vaginalis, and Mycoplasma genitalium.

Patients who are diagnosed with gonorrhea should also be treated for which of the following infections? A) Chancroid B) Chlamydia trachomatis C) Herpes genitalis D) Pelvic inflammatory disease (PID)

B) Chlamydia trachomatis When diagnosed with gonorrhea, the patient should also be treated for Chlamydia trachomatis.

There is a higher risk of balanitis in which of the following conditions? A) Renal insufficiency B) Diabetes mellitus C) Graves' disease D) Asthma

B) Diabetes mellitus Balanitis is a yeast infection of the glans of the penis. Men who are not circumcised and who have diabetes mellitus are at higher risk for developing balanitis.

A patient complains of twisting his right knee while walking that morning. The knee is swollen and tender to palpation. The nurse practitioner diagnoses a grade II sprain. The initial treatment plan includes which of the following? A) Intermittent application of cold packs the first 24 hours followed by applications of low heat at bedtime B) Elevation of the affected limb and intermittent applications of cold packs for the next 48 hours C) Rechecking the knee in 24 hours and encouraging isometric exercises D) Application of an elastic bandage wrap to the affected knee

B) Elevation of the affected limb and intermittent applications of cold packs for the next 48 hours Elevation of the injured knee above the heart will reduce the amount of swelling that can occur. Use of ice packs immediately after the injury is most effective and will reduce swelling in the tissue. Ice the affected area for 15 to 20 minutes at a time intermittently to prevent frostbite and further damage to tissue. Allowing 30 to 45 minutes between icing of the limb is recommended.

Which of the following viral infections is associated with occasional abnormal forms of lymphocytes during an acute infection? A) Cytomegalovirus (CMV) B) Epstein-Barr virus (EBV) C) Human papillomavirus (HPV) D) Coxsackievirus

B) Epstein-Barr virus (EBV) EBV is a member of the herpesvirus family and one of the most common viruses. During adolescence, EBV causes infectious mononucleosis. In most cases of infectious mono, the clinical diagnosis can be made from the triad of fever, pharyngitis, and lymphadenopathy lasting 1 to 4 weeks. Serology tests show normal to moderately elevated white blood cells and increased numbers of lymphocytes, greater than 10% atypical lymphocytes, and a positive reaction to a mono spot test. The antibody response in primary EBV infection appears to be quite rapid.

A newborn infant who is small for gestational age is noted to have shortened palpebral fissures and microcephaly with a small jaw. This infant is most likely to be diagnosed with: A) Down syndrome B) Fetal alcohol syndrome C) Growth retardation D) Hydrocephalus

B) Fetal alcohol syndrome Classic symptoms of fetal alcohol syndrome include small palpebral fissures and microcephaly with a small jaw.

What does a potassium hydroxide (KOH) prep help the nurse practitioner diagnose? A) Herpes zoster infections B) Fungal infections C) Herpes simplex infections D) Viral infections

B) Fungal infections The potassium hydroxide (KOH) prep test is performed to evaluate for tinea or candida (yeast) infection of the skin. In vaginal discharge, the yeast organism is outside the skin cells, so KOH is not needed to visualize it. But for skin cells, yeast is not visible unless the skin cell walls are destroyed by KOH. The test involves placing a sample of skin on a glass slide, with one to two drops of KOH (causes lysis of skin cells) and a coverslip on top. If done correctly, you can visualize the budding spores and pseudohyphae.

A high school teacher complains of a dry cough for the past 6 weeks. It worsens when he is supine. He has episodes of heartburn, which he self-treats with an over-the-counter (OTC) antacid. He chews mints for his "bad breath." Which of the following is a possible cause for this patient's cough? A) Asthma B) Gastroesophageal reflux C) Pneumonia D) Chronic postnasal drip

B) Gastroesophageal reflux Classic signs of gastroesophageal reflux disease (GERD) include acid reflux (regurgitation) into the esophagus, heartburn, and nausea. Complications include ulcers, esophageal strictures, Barrett's esophagus, cough, asthma, and throat or laryngeal inflammation. Risk factors include obesity, pregnancy, smoking, and alcohol use.

A newborn's mother is discovered to be HBsAg (hepatitis B surface antigen) positive. Which of the following would you recommend for this infant? A) Give the infant hepatitis B immunoglobulin B) Give the infant both hepatitis B vaccine and hepatitis B immunoglobulin C) Give the infant hepatitis B vaccine only D) Send the infant home because he is not infected

B) Give the infant both hepatitis B vaccine and hepatitis B immunoglobulin For a mother who tests positive for HBsAg, the newborn infant should be given hepatitis B vaccine and hepatitis B immunoglobulin for protection.

Which of the following drug classes is indicated for initial treatment of an uncomplicated case of Helicobacter pylori-negative peptic ulcer disease? A) Proton-pump inhibitors B) H2 receptor antagonists C) Antibiotics D) Antacids

B) H2 receptor antagonists Because the ulcer is not infected with Helicobacter pylori, antibiotics are not recommended. The first-line treatment option are H2 receptor antagonists (also known as H2 blockers) such as ranitidine (Zantac), famotidine (Pepcid), or nizatidine (Axid). Other causes of peptic ulcer disease are nonsteroidal anti-inflammatory drugs (NSAIDs); the patient should be educated to avoid use of these agents.

Which of the following human papillomavirus (HPV) strains is associated with cervical cancer? A) HPV subtypes 12 and 14 B) HPV subtypes 16 and 18 C) HPV subtypes 20 and 26 D) HPV subtype 30

B) HPV subtypes 16 and 18 Human papillomavirus (HPV) types 16 and 18 are highrisk oncogenic strains. If an HPV-typing test shows one or both strains, the next step is to refer the patient for colposcopy and biopsy. About 70% of cervical cancers worldwide are due to types 16 and 18 (Centers for Disease Control and Prevention, 2014). HPV types that are low risk cause warts in the cervix, vagina, vulva, penis, anus, and rectum.

An 8-year-old child is seen as a walk-in appointment by the nurse practitioner. The mother reports that her child has been febrile for 2 days and is not eating well due to painful sores inside the mouth. The child's temperature is 101°F, pulse is 88 beats/min, and respirations are 14 breaths/min. During the physical exam, the nurse practitioner notices several small blisters and shallow ulcers on the child's pharynx and the oral mucosa. The child has small round red rashes on both palms and soles. Which of the following conditions is most likely? A) Herpes simplex infection B) Hand, foot, and mouth disease (HFMD) C) Varicella infection D) Secondary syphilis infection

B) Hand, foot, and mouth disease (HFMD) HFMD is caused by the coxsackievirus. The virus is found in the saliva, sputum, nasal mucus, feces, and blister fluid. It is transmitted through direct contact of the secretion or in fomites (e.g., preschool toys). Treatment is symptomatic.

During a well child visit, a new father wants to know whether he can give fresh whole milk to his 6-month-old son. The nurse practitioner would recommend that: A) He can start giving whole milk but not skim milk by 6 months of age B) He should not give whole milk to his son until the boy is at least 12 months of age C) He can give whole milk to his son at anytime D) He should not give whole milk to his son without diluting it with water

B) He should not give whole milk to his son until the boy is at least 12 months of age The American Academy of Pediatrics does not recommend offering whole milk to children younger than 12 months of age.

The mother of a 16-year-old boy is concerned that her son is not developing normally. On physical exam, the patient is noted to have small testes with no pubic or facial hair. What is the most appropriate statement to the mother? A) Her son is developing normally B) Her son's physical development is delayed and should be evaluated by a pediatric endocrinologist C) Her son should be rechecked in 3 months; if he still does not have secondary sexual characteristics, a thorough hormonal workup should be initiated D) Her son's physiological development is slower than normal but is within the lower limit of normal for his age group

B) Her son's physical development is delayed and should be evaluated by a pediatric endocrinologist Puberty may be delayed for several years and still occur normally, in which case it is considered constitutional delay, a variation of healthy physical development. Delay of puberty may also occur due to malnutrition, many forms of systemic disease, or to defects of the reproductive system (hypogonadism) or the body's responsiveness to sex hormones. Hypogonadism occurs when the sex glands produce little or no hormones. In men, these glands (gonads) are the testes. A 16-year-old male without secondary sexual characteristics should be referred to an endocrinologist. If there is no testicular development by 14 years of age, an endocrinology consult is warranted.

What does a positive posterior drawer sign in a 10-year-old soccer player signify? A) Normal knee B) Instability of the knee C) Swelling on the knee D) Injury to the meniscus

B) Instability of the knee The drawer test is used to identify mediolateral or anteroposterior plane instability of the knee. The test is performed on the unaffected and affected knee for comparison. The anterior drawer test evaluates the anterior cruciate ligament (ACL). To perform the test, the patient lies supine and the knee is placed at 90-degree flexion. Grasp the posterior aspect of the tibia over the upper calf muscle; then, with a steady force, try to push the lower leg forward and backward. Anterior or posterior movement of the knee is positive. With the leg extended, stabilize the femur with one hand and the ankle with the other. Try to abduct and adduct the knee. There should be no medial or lateral movement.

An 8-month-old girl is brought by her grandmother to see the nurse practitioner because of intermittent, random episodes of vomiting, abdominal bloating, currant jelly stools, and irritability with poor appetite. The infant is trending in the 10th percentile on the growth chart and appears lethargic. During the abdominal exam, a sausage-like mass is palpated on the right side of the abdomen. The infant's presentation is highly suggestive of which condition? A) Lactic intolerance B) Intussusception C) Inflammatory bowel disease D) Irritable bowel syndrome

B) Intussusception The "classic triad" of intussusception—currant jelly stools, sausage- like mass, and pain (an infant with inconsolable crying, who draws up knees to abdomen)—is seen in less than 15% of patients. A sausage-shaped abdominal mass may be palpated on the right side of the abdomen. One third of patients do not pass blood or mucus or develop an abdominal mass. Older children may present with pain alone (Kitagawa & Miqdady, 2016).

Which of the following would be appropriate initial management of a second-degree burn? A) Irrigate with hydrogen peroxide and apply Silvadene cream BID B) Irrigate with normal saline and apply Silvadene cream BID C) Irrigate with tap water and apply Neosporin ointment BID D) Unroof all intact blisters and apply antibiotic ointment BID

B) Irrigate with normal saline and apply Silvadene cream BID Burns should be cleansed with saline solution and Silvadene (silver sulfadiazine) cream applied BID to the site. Hydrogen peroxide is no longer recommended. Intact blisters should not be unroofed.

All of the following are false statements about atopic dermatitis except: A) Contact with cold objects may exacerbate the condition B) It does not have a linear distribution C) It is associated with bullae D) The lesions have vesicles that are full of serous exudate

B) It does not have a linear distribution Atopic dermatitis, also known as eczema, is a skin condition in which the lesions occur in a linear fashion. They may have many different stages, including erythematous papules and vesicles, with weeping, drainage, and/or crusting. Lesions are commonly pruritic and are found on the scalp, face, forearms, wrists, elbows, and backs of the knees.

A kindergarten teacher is diagnosed with acute streptococcal pharyngitis. On exam, her throat is a bright-red color with no tonsillar exudate, and clear mucus is seen on the lower nasal turbinates. The urinalysis shows a large amount of white blood cells and is positive for nitrites. The patient has a sulfa allergy and thinks she is also allergic to penicillins. Which of the following is the best treatment choice? A) Amoxicillin-clavulanic acid (Augmentin) 500 mg PO BID B) Levofloxacin (Levaquin) 250 mg PO daily C) Trimethoprim-sulfamethoxazole (Bactrim DS) 1 tablet PO BID D) Clarithromycin (Biaxin) 500 mg PO BID

B) Levofloxacin (Levaquin) 250 mg PO daily Streptococcal pharyngitis and urinary tract infection are both covered by using Levaquin, which is a fluoroquinolone. Amoxicillin-clavulanic acid (Augmentin) and trimethoprim-sulfamethoxazole (Bactrim) would not be used because of the patient's allergies. Fluoroquinolones can be used in patients aged 18 years or older.

The gold-standard test for visualizing a torn meniscus or joint abnormalities is the: A) Computed tomography (CT) scan B) Magnetic resonance imaging (MRI) scan C) X-ray with special views of the affected knee D) Lachman's maneuver

B) Magnetic resonance imaging (MRI) scan MRIs provide good visualization of soft tissues of the body (most cancers, brain, cartilage, muscles, inflammation, etc.). They are best used to evaluate tissues with high water content. Patients with metal implants, such as cochlear implants and cardiac pacemakers, should be carefully screened. The MRI does not use radiation, but uses strong magnetic and radio waves to visualize body structures.

A 22-year-old man is brought to an urgent care center by his anxious mother. She reports that her son returned from a camping trip 2 days ago with a high fever and bad headache. Apparently, he had complained to her of a painful and stiff neck along with nausea shortly after he returned. The mother states that her son started breaking out in a rash the day before, parts of which are now turning a dark-red to purple color. During the physical exam, the nurse practitioner evaluates the patient for Kernig's sign, which is positive. Which of the following conditions is most likely? A) Stevens-Johnson syndrome B) Meningococcemia C) Rocky Mountain spotted fever D) Erythema multiforme

B) Meningococcemia Kernig's maneuver is performed by flexing both hips and legs and having the patient straighten the legs against resistance, testing for meningitis. A positive test indicates meningitis. Other characteristics of meningitis include high fever, headache, stiff neck, and nausea/vomiting.

A 16-year-old girl presents in the school clinic and tells the nurse that she would like birth control pills for contraception because she is sexually active. How should the nurse practitioner proceed? A) Refuse to see the patient until consent can be obtained from her parent or legal guardian B) Perform a history and physical exam and, if the patient does not have health contraindications, prescribe birth control pills C) Speak with the patient about safe sex and have her obtain parental consent for the physical exam D) Perform a gynecological exam and obtain a Pap smear

B) Perform a history and physical exam and, if the patient does not have health contraindications, prescribe birth control pills In the United States, the majority of states grant minors the right to privacy to obtain contraceptives (such as birth control pills), pregnancy testing, and testing for sexually transmitted infection (Guttmacher Institute, n.d.). To safely prescribe birth control pills, it is necessary to obtain a history and perform a physical exam to rule out health contraindications, rule out pregnancy, and evaluate the patient's decision-making skills and ability to follow instructions. Contraceptive options and safe sex education are also part of the first visit. It is not necessary to obtain a Pap smear to prescribe birth control pills. The U.S. Preventive Services Task Force does not recommend a Pap smear for woman younger than age 21 years.

During a physical exam of a 6-year-old child, you note some pitting on the fingernails. This finding is correlated with: A) Iron-deficiency anemia B) Psoriasis C) Onychomycosis D) Vitamin C deficiency

B) Psoriasis Fingernail pitting is correlated with psoriasis. Psoriasis can cause pitting on all finger and toenails, along with thickening and an irregular shape of the nail.

A 28-year-old woman with a history of hypothyroidism presents to an urgent care clinic complaining of numbness and tingling in the fingertips of both her hands for several hours. On examination, both radial pulses are at +2 and equal bilaterally. The patient reports that over the past few months she has had identical episodes, each lasting several hours. During these episodes, the skin changes color from blue to white, and then to dark red. Eventually, it returns to normal and the tingling and numbness disappear. Which of the following conditions is best described? A) Hashimoto's disease B) Raynaud's phenomenon C) Peripheral neuropathy D) Vitamin B12 deficiency anemia

B) Raynaud's phenomenon Raynaud's phenomenon involves an interruption in the blood flow to fingers and toes (sometimes nose and ears), due to spasms in the blood vessels. During a Raynaud's attack, the affected area typically turns white, and, as oxygen fails to reach the extremities, they can turn blue, tingle, or throb painfully, and the affected area may swell. Symptoms may resolve quickly or last for hours. Raynaud's disease is more common in women, is not curable, and it is associated with increased risk of other autoimmune diseases such as rheumatoid arthritis or lupus, but treatment can decrease symptoms. The preferred drug is a calcium channel blocker such as nifedipine (Norvasc) or amlodipine (Procardia).

A 10-year-old boy complains of sudden onset of scrotal pain upon awakening that morning. He is also complaining of severe nausea and vomiting. During the physical examination, the nurse practitioner finds a tender, warm, and swollen left scrotum. The cremasteric reflex is negative and the urine dipstick is negative for leukocytes, nitrites, and blood. What type of follow-up should this patient receive? A) Refer him to a urologist within 48 hours B) Refer him to the emergency department as soon as possible C) Prescribe ibuprofen (Advil) 600 mg QID for pain D) Order a testicular ultrasound for further evaluation

B) Refer him to the emergency department as soon as possible Immediate referral to the emergency department is required to prevent irreversible ischemia. Success of treatment is usually 100% if treated within the first 6 hours and 0% if treated after 24 hours. The diagnosis of testicular torsion is often made clinically, but if it is in doubt, an ultrasound is helpful in evaluating the condition. Emergency diagnosis and treatment are usually required within 4 to 6 hours to prevent necrosis.

Which type of exercise would you recommend to a 65-year-old arthritic patient who complains of a new onset of a painful, swollen left knee caused by overworking in the garden for 2 days? A) Quadriceps-strengthening exercises of the left knee followed by the application of cold packs for 20 minutes four times a day B) Rest the joint and apply cold packs intermittently for the next 48 hours C) Passive range of motion and cold packs D) A cool tub bath with warm packs on the knee to avoid stiffening of the joint

B) Rest the joint and apply cold packs intermittently for the next 48 hours New onset of a painful, swollen left knee (inflammation) should be treated using RICE: Rest the knee/joint, use alternating ice packs for the first 24 to 48 hours, use compression if knee feels unstable, and elevate the limb to decrease swelling.

A 15-month-old who is eating and behaving normally is found to have a high fever. After a few days, the fever resolves and the child breaks out in a maculopapular rash. This is a description of which of the following conditions? A) Erythema infectiosum B) Roseola infantum C) Fifth disease D) Scarlet fever

B) Roseola infantum Roseola infantum is caused by herpes virus 6 (HHV-6). The signs and symptoms of roseola infantum include high fever for a few days with a maculopapular rash occurring after the fever breaks. Febrile seizures occur in up to 15% of infants. Fifth disease (erythema infectiosum) is caused by parvovirus B19. It causes a red facial rash or "slapped cheek" appearance; in a few days a lacy-appearing rash appears on the trunk, arms, and legs. Erythema infectiosum is the same as fifth disease. Scarlet fever is a red rash that feels like sandpaper that usually begins on the neck and trunk and spreads to the extremities. Patients with scarlet fever have other symptoms such as fever, sore throat, and beefy red tongue. Scarlet fever is caused by the group A Streptococcus bacterium.

The mother of a 12-month-old infant reports to the nurse practitioner that her child had a high fever for several days, which spontaneously resolved. After the fever resolved, the child developed a maculopapular rash. Which of the following is the most likely diagnosis? A) Fifth disease (erythema infectiosum) B) Roseola infantum (exanthema subitum) C) Varicella D) Infantile maculopapular rashes

B) Roseola infantum (exanthema subitum) Roseola infantum is a common viral rash that is caused by the human herpesvirus. The most common ages of onset are between 6 months and 2 years. The rashes are maculopapular (small round pink-colored) rashes that first appear on the trunk and then spread to the extremities.

Koplik's spots are associated with: A) Poxvirus infections B) Rubeola C) Kawasaki's disease D) Rubella

B) Rubeola Rubeola is the Latin name for measles. Signs and symptoms of measles infection include fever over 101°F, coryza, cough, conjunctivitis, rash, and Koplik's spots on the buccal mucosa. Do not confuse rubeola with rubella (German measles or 3-day measles).

A 16-year-old complains of a severe sore throat for 3 days along with a generalized rash and fever. The skin has the texture of fine sandpaper. This constellation of findings best describes: A) Kawasaki's disease B) Scarlatina C) German measles D) Rubeola

B) Scarlatina Scarlatina (scarlet fever) is a rash that usually first appears on the neck and chest, then spreads over the body. It is described as "sandpapery" in feel. The texture of the rash is more important than the appearance in confirming the diagnosis. The rash can last for more than a week. As the rash fades, peeling (desquamation) may occur around the fingertips, toes, and groin area. Another sign is a bright red tongue with a "strawberry" appearance.

You would advise an 18-year-old female student who has been given a booster dose of MMR at the college health clinic that: A) She might have a low-grade fever during the first 24 to 48 hours B) She should not get pregnant within the next 4 weeks C) Her arm will be very sore at the injection site for 24 to 48 hours D) Her arm will have some induration at the injection site in 24 to 48 hours

B) She should not get pregnant within the next 4 weeks MMR should not be administered to women known to be pregnant. In addition, women should be counseled to avoid becoming pregnant for 28 days following vaccination.

A college freshman who is using oral contraceptives calls the nurse practitioner's office asking for advice. She forgot to take her pills 2 days in a row during the second week of the pill cycle and wants to know what to do. What is the best advice? A) Start a new pack of pills and dispose of the old one B) Take two pills today and two pills tomorrow, and have your partner use condoms for the rest of the pill cycle C) Stop taking the pills right away, and start a new pill cycle in 2 weeks D) Take one pill today and two pills tomorrow, and have your partner use condoms

B) Take two pills today and two pills tomorrow, and have your partner use condoms for the rest of the pill cycle When forgetting to take the birth control pill on 2 consecutive days, it is recommended to take two pills immediately and two pills the next day, then continue the rest of the pack. Stress the importance of the use of condoms for protection against pregnancy and STIs.

A 13-year-old adolescent girl is brought to the health clinic by her mother for a sports physical. The mother reports that the teen's last vaccines were given at the age of 6 years. Which of the following vaccines is recommended by the Centers for Disease Control and Prevention (CDC) for this patient? A) Td and HPV vaccines B) Tdap, MCV4, and the HPV vaccines C) DTap and the flu vaccine D) DT and MCV4 vaccines

B) Tdap, MCV4, and the HPV vaccines Vaccine questions usually are not this complicated, but there are several lessons that can be learned with this question. The 2017 Centers for Disease Control and Prevention (CDC) recommendations for individuals 13 to 18 years of age are the Tdap catch-up (if not received at age 11-12 years), human papillomavirus (HPV) catch-up (if not received at age 11-12 years), and the MCV4 or meningococcal conjugate vaccine (Menactra). Only one dose of Tdap is recommended (lifetime). Thereafter, the Td form of the vaccine is indicated every 10 years. The DTaP (diphtheria-tetanus-acellular pertussis) and DT (diphtheria-tetanus) forms of the tetanus vaccine are not given after the age of 7 years.

An 18-year-old female patient is being followed up for acne by the nurse practitioner. During the facial exam, papules and pustules are noted mostly on the forehead and the chin areas. The patient has been using prescription topical medications and over the-counter medicated soap daily for 6 months without much improvement. Which of the following would the nurse practitioner recommend next? A) Isotretinoin (Accutane) B) Tetracycline (Sumycin) C) Clindamycin topical solution (Cleocin T) D) Minoxidil (Rogaine)

B) Tetracycline (Sumycin) First-line treatment for acne vulgaris includes over-the counter medicated soap and water with topical antibiotic gels. The next step in treatment would be the initiation of oral tetracycline.

All of the following are true statements about the human papillomavirus vaccine (Gardasil) except: A) The Centers for Disease Control and Prevention (CDC) recommends the first dose at age 11 to 12 years B) The CDC does not recommend the HPV vaccine for males C) For children younger than age 14 years, only two doses of the vaccine are needed D) The minimum age at which the vaccine can be given is 9 years

B) The CDC does not recommend the HPV vaccine for males The HPV vaccine is now recommended for both males and females. If it is given before age 14 years, only two doses are required, But if the child is older than 14 years, three doses are required to be given over 6 months. The vaccine can be given until the age of 26 years, especially if the individual is at high risk. Do not use the vaccine in children younger than 9 years. There are two types of HPV vaccine (Gardasil and Cervarix). Gardasil can be used for both genders, but Cervarix can only be used for females.

Which of the following is a true statement regarding pes planus in an infant? A) It should be evaluated by a pediatric orthopedist if spontaneous correction does not occur by age 12 months B) The fat pads on an infant's feet can mimic pes planus C) It is always corrected by wearing special orthotic shoes D) It is also called talipes equinovarus

B) The fat pads on an infant's feet can mimic pes planus The fat pads on an infant's feet can resemble pes planus, or flat feet.

The mother of a 4-month-old girl calls your office and reports that the infant has a fever of 101.4°F. The infant received her immunizations yesterday. Which of the following is correct? A) The fever is most likely due to the combination of the MMR and polio vaccines B) The fever is most likely due to the pertussis component of the DTaP vaccine C) The infant is probably starting a viral upper respiratory infection D) The infant had an allergic reaction to one of the vaccines given and should be brought to the emergency department

B) The fever is most likely due to the pertussis component of the DTaP vaccine The pertussis component of the vaccine is most likely the cause of the fever.

The following statements are all true regarding herpes zoster except: A) It is due to reactivation of latent varicella virus B) The typical lesions are bullae C) It is usually more severe in immunocompromised individuals D) Infection of the trigeminal nerve ophthalmic branch can cause corneal blindness

B) The typical lesions are bullae Herpes zoster (shingles) occurs secondary to reactivation of the varicella-zoster (VZV) virus. This infection can be more severe in immunocompromised patients due to their inability to fight infection. Shingles rash starts as small blisters on a red base, with new blisters continuing to form for 3 to 5 days. The rash is painful, and commonly appears in clusters, following one dermatome on one side of the body. When the trigeminal nerve is involved, there is an increased risk of corneal blindness.

While reviewing some lab reports, the nurse practitioner notes that one of the results for her teenage male patient is abnormal. The liver function tests are all normal except for a slight elevation in the alkaline phosphatase level. The patient is a member of a soccer team and denies any recent injury. Which of the following statements is true? A) It is an indication of possible liver damage from alcohol; order a liver ultrasound to rule out fatty liver B) This is a normal finding due to the skeletal growth spurt in this age group C) The patient needs to be evaluated further for pancreatic disease D) The patient needs an ultrasound of the liver to rule out fatty liver and referral to a pediatric rheumatologist

B) This is a normal finding for his age group Alkaline phosphatase is part of a group of related enzymes. The bone form of the enzyme creates the alkaline conditions it requires to be most active with a chemical reaction involving the osteoblasts. The rapid bone growth and increased deposit of calcium during growth spurts and adolescence elevates the alkaline phosphatase level.

What type of cast is used to stabilize a fracture of the scaphoid bone? A) Long arm cast B) Thumb spica cast C) Short leg cast D) Leg cylinder cast

B) Thumb spica cast The scaphoid bone is located on the radial aspect of the wrist (same side as the thumb). Another term for scaphoid fracture is navicular space fracture of the wrist. In patients with this injury, palpating in the anatomic snuffbox area reveals local tenderness. A thumb spica cast is used to stabilize fractures of the wrist. It is applied below the elbow (forearm in neutral position) and extends to the hand (wrist and thumb are immobilized). Sometimes, this type of fracture is stabilized with a screw (surgery) because it has a higher rate of nonunion.

Rocky Mountain spotted fever is caused by the bite of a: A) Mosquito B) Tick C) Louse D) Flea

B) Tick Rocky Mountain spotted fever is caused by a bite from a tick infected with the parasite Rickettsia rickettsii. The mortality rate is 1% to 7% if left untreated.

A 25-year-old woman presents with severe right-sided pelvic pain that began 48 hours ago. She reports small amounts of vaginal bleeding. The pain is aggravated by jumping or any movement that jars her pelvis. The best initial intervention is which of the following? A) Follicle-stimulating hormone (FSH) test B) Urine human chorionic gonadotropin (hCG) test C) Pelvic ultrasound D) CBC with white cell differentials

B) Urine human chorionic gonadotropin (hCG) test Severe right-sided pain with vaginal bleeding in a 25-year-old patient requires an evaluation for pregnancy by initially performing a urine hCG test, which can be done quickly in the primary care clinic. A serum hCG test can also be drawn at the same visit. If a ruptured ectopic pregnancy is suspected, call 911.

You have diagnosed a 30-year-old male patient with contact dermatitis on the left side of the face secondary to poison ivy. You would recommend: A) Washing with antibacterial soap BID to reduce risk of secondary bacterial infection until it is healed B) Zanfel poison ivy wash C) Clotrimazole (Lotrimin) cream BID for 2 weeks D) Halcinonide (Halog) 1% ointment BID for 2 weeks

B) Zanfel poison ivy wash Zanfel is a soap-like product (OTC) that removes urushiol oil from poison ivy, poison sumac, and poison oak. It will relieve the itch and pain quickly. A topical steroid can be used to speed up healing. For rashes, hydrocortisone cream 1% BID (OTC) is helpful.

What is the best description of Cullen's sign? A) The onset of hyperactive bowel sound before the onset of ileus B) A reddish-purple discoloration that is located on the flank area C) A bluish discoloration or bruising that is located on the umbilical area D) The acute onset of subcutaneous bleeding seen during acute pancreatitis

C) A bluish discoloration or bruising that is located on the umbilical area Cullen's sign is the acute onset of bluish discoloration that is located on the umbilical/periumbilical area, caused by bruising underneath the skin. A bluish discoloration located on the flank area is called the Grey-Turner's sign. It is a sign of a severe case of pancreatitis.

The following children are considered at higher risk for tuberculosis (TB) except: A) A child who has recently been diagnosed with leukemia B) An infant whose family is homeless C) A child who was born in Japan D) A Hispanic child with asthma who is using a steroid inhaler

C) A child who was born in Japan Japan is not considered a high-risk country for tuberculosis (TB) infection; other Asian countries, such as India, Bangladesh, Pakistan, China, and the Philippines, have a higher incidence. Other high-risk areas are Africa, the Western Pacific, and Europe (Russia). Additional risk factors for TB are immunocompromised status (HIV, steroid therapy), homelessness, injection drug users, and working or residing with people at high risk for TB.

Which of the following should you expect to find on a wet-mount slide of a patient diagnosed with bacterial vaginosis? A) Tzanck cells B) A large number of leukocytes and squamous epithelial cells C) A large number of squamous epithelial cells whose surfaces and edges are coated with large numbers of bacteria along with a few leukocytes D) Epithelial cells and a small amount of blood

C) A large number of squamous epithelial cells whose surfaces and edges are coated with large numbers of bacteria along with a few leukocytes Diagnosis of bacterial vaginosis includes three of four Amsel criteria: (1) white, thick adherent discharge; (2) pH greater than 4.5; (3) positive whiff test (amine odor mixed with 10% potassium hydroxide [KOH]); (4) clue cells greater than 20% on a wet mount (epithelial cells dotted with large numbers of bacteria that obscure cell borders).

The following skin findings are considered macules except: A) A freckle B) Petechiae C) Acne D) A flat, 0.5-cm brown birthmark

C) Acne A macule is a flat, nonraised lesion on the skin. Acne lesions are papules because they consist of raised, erythemic lesions on the skin. A freckle, petechiae, and a flat birthmark are all considered macules.

A new mother reports to you that her 6-month-old infant has a cold and has a fever of 99.8°F. The infant is not irritable and is feeding well without problems. The mother wants to know whether it is okay for him to be immunized at this time. Which of the following statements is true? A) The infant should not be immunized until he is afebrile B) An infant with a cold can be immunized at any time C) An infant with a cold can be immunized as long as the infant's temperature is no higher than 100.4°F D) Because immunization is so important, it should be given to the infant as scheduled

C) An infant with a cold can be immunized as long as the infant's temperature is no higher than 100.4°F Immunization is acceptable as long as the temperature is not higher than 100.4°F An infant with a cold can be immunized as long as the infant's temperature is no higher than 100.4°F.

A positive anterior drawer sign of the knee is highly suggestive of? A) Meniscus damage B) Posterior cruciate ligament damage C) Anterior cruciate ligament damage D) Medial collateral ligament damage

C) Anterior cruciate ligament damage A positive anterior drawer sign occurs when there is damage to the anterior cruciate ligament. The patient is tested in supine position with knees flexed at 90 degrees and feet flat on the exam table. Hold the proximal lower leg (upper tibia) and push the leg anteriorly. The test is positive if the affected knee slides or has laxity when compared with the uninjured knee. Refer patient to an orthopedic specialist if the knee is unstable. The best imaging test for joints is the MRI.

The nurse practitioner suspects that a middle-aged woman may have systemic lupus erythematosus (SLE). Which of the following laboratory tests is most specific for this disease? A) Erythrocyte sedimentation rate (ESR) B) C-reactive protein (CRP) C) Antinuclear antibody (ANA) D) IgG antibody

C) Antinuclear antibody (ANA) The ANA test is usually positive in lupus patients. Other types of autoantibody testing recommended for these patients, in addition to ANA tests, are antiphospholipid antibodies, antibodies to double-stranded DNA, and anti-Smith (Sm) antibodies. Patients with suspected lupus should be referred to a rheumatologist. The erythrocyte sedimentation rate (ESR) and the C-reactive protein (CRP) are nonspecific findings of inflammation and are elevated in patients with autoimmune diseases, infections, and others.

A 21-year-old college student has recently been informed that he has a human papillomavirus (HPV) infection on the shaft of his penis. Which of the following methods can be used to visualize subclinical HPV lesions on the penile skin? A) Perform a KOH (potassium hydroxide) exam B) Scrape off some of the affected skin and send it for a culture and sensitivity C) Apply acetic acid to the penile shaft and look for acetowhite changes D) Order a serum herpesvirus titer

C) Apply acetic acid to the penile shaft and look for acetowhite changes Lesions of human papillomavirus (HPV) infection will turn white with application of acetic acid. Routine use of this procedure to detect mucosal changes due to HPV is not recommended because results do not influence clinical management (CDC, 2015). Condoms may lower the risk of HPV infection and might decrease the time to clear in women with HPV infection.

A 16-year-old male patient with psoriasis is scratching and rubbing one of the psoriatic plaques located on his right elbow. Fine silvery scales with pinpoint areas of bleeding are noted on the plaque. What is the name of this clinical finding? A) Erosion B) Lichenification C) Auspitz sign D) Koebner phenomenon

C) Auspitz sign Auspitz sign is observed when a psoriatic lesion is scraped or scratched multiple times, resulting in pinpoint areas of bleeding on the psoriatic plaque. Lichenification is thickened skin caused by chronic irritation (itching). It is considered a secondary lesion. Another type of secondary skin lesion is an erosion, which occurs with loss of epidermis (but not the dermis). It can resemble a shallow ulcer. The Koebner phenomenon refers to lesions (often linear) that appear on the skin of patients with psoriasis as a result of trauma to the skin (e.g., scratching).

Infection of the skin and mucous membranes with human papillomavirus (HPV) is associated with an increased risk of certain types of cancers. All of the following cancers are associated with HPV infection except: A) Oropharyngeal cancer B) Anal cancer C) Basal cell carcinoma D) Penile cancer

C) Basal cell carcinoma Basal cell carcinoma is caused by chronic ultraviolet (UV) skin damage (i.e., by sunlight), not human papillomavirus (HPV) infection. It is the most common type of skin cancer. HPV infection increases the risk of precancer and cancer of the oropharynx/larynx, anus, penis, and cervix.

Candidal intertrigo is the name for an infection that is caused by the yeast Candida albicans. What is the location of this type of candida infection? A) Scalp B) Flexor areas of the elbows and the knees C) Body areas where skin rubs together, such as under breasts or in groin area D) Hands

C) Body areas where skin rubs together, such as under breasts or in groin area Candidal intertrigo infections are more common in obese individuals and in women with pendulous breasts. It is found in areas where skin rubs against skin (under breasts, in the groin area, and on stomach folds in the obese). It is more common in warm and humid weather (summer).

A new mother is breastfeeding her full-term 4-week-old infant. She wants to know whether she should give the infant vitamin supplements. The best advice is: A) Because she is breastfeeding, the infant does not need any vitamin supplements B) Breast milk gives the infant all the vitamins he needs until 12 months of age C) Breastfed infants require vitamin D supplementation beginning in the first few days of life D) Infant formula can be used to supplement breastfeeding as needed

C) Breastfed infants require vitamin D supplementation beginning in the first few days of life The American Academy of Pediatrics (2010) recommends vitamin D 400 IU/day supplementation starting at the first few days of life for all breastfed infants to decrease the risk of rickets. It can be given as part of a multivitamin infant supplement or in a vitamin D supplement. Both are over the counter. Breastmilk alone does not provide infants with an adequate intake of vitamin D (CDC, 2015).

A first-grader presents to a school nurse practitioner with a few blisters on one arm and on his face. The child keeps scratching the affected areas. Some of the lesions have ruptured with yellow serous fluid that crusts easily. These findings best describe: A) Acute cellulitis B) Herpes zoster C) Bullous impetigo D) Erysipelas

C) Bullous impetigo Bullous impetigo is an infection of the skin caused by Staphylococcus aureus, which produces exfoliative toxin A. The infection is more common in young children. It is characterized by flaccid large blisters filled with serous fluid. When the bullae rupture, the serous fluid dries up and resembles honey-colored crusts. If the child has a limited number of skin lesions, topical therapy with mupirocin (Bactroban) is effective. But if lesions are extensive, systemic antibiotics, such as Dicloxacillin and cephalexin, are the treatment of choice.

Balanitis is caused by: A) Staphylococcus aureus B) Streptococcus pyogenes C) Candida albicans D) Trichomonads

C) Candida albicans Balanitis is a yeast infection of the glans of the penis. Candida albicans is the causative source. Staphylococcus aureus and Streptococcus pyogenes are bacterial infections. Trichomonads are protozoans that cause infection.

Human papillomavirus infection in women has been associated with the development of: A) Ectopic pregnancy B) Infertility C) Cervical cancer D) Pelvic inflammatory disease

C) Cervical cancer There are more than 30 types of human papillomavirus (HPV). HPV types in the anogenital region have been strongly associated with low-grade and high-grade cervical change, cervical neoplasia, as well as anogenital and other cancers. An HPV vaccine is recommended for girls and boys between the ages of 15 and 21 years of age to help prevent four strains of this virus.

All of the following may help relieve the symptom(s) of gastroesophageal reflux disease (GERD) except: A) Losing weight B) Stopping caffeine intake C) Chewing breath mints D) Stopping alcohol intake

C) Chewing breath mints Gastroesophageal reflux disease (GERD) is a condition in which food comes up the esophagus from the stomach because of a weak sphincter. The reflux is usually worsened by lying down, and can cause a cough and esophageal irritation if not treated. Effective treatment may include weight loss, decreased caffeine intake, and avoidance of alcohol.

A patient diagnosed with bacterial vaginosis should be advised that her sexual partner: A) Needs treatment with ceftriaxone (Rocephin) 250 mg IM with doxycycline 100 mg BID for 14 days B) Needs treatment with metronidazole (Flagyl) 500 mg PO BID for 7 days and 1 dose of azithromycin (Zithromax) C) Does not need treatment D) Needs treatment with clotrimazole cream (Lotrimin) on his penis BID for 1 to 2 weeks

C) Does not need treatment Bacterial vaginosis is a bacterial infection, but is not considered a sexually transmitted disease for which the partner needs treatment. Studies show that men rarely carry this infection.

At what time of the day would you recommend a scotch tape test be done to evaluate for a suspected case of enterobiasis? A) In the evening after dinner B) At night before bed C) Early in the morning D) It does not matter what time of the day the test is done

C) Early in the morning The scotch tape test is best done in the morning for several days in a row. The female pinworms do not lay eggs every day; therefore, testing over the course of several days will be more accurate.

The ELISA and Western blot tests are used to detect which of the following? A) HIV RNA B) HIV DNA C) HIV antibodies D) HIV antigen

C) HIV antibodies The ELISA (enzyme-linked immunosorbent assay) and Western blot tests detect only HIV antibodies. That is why there is a "window period" with HIV infection. If the test is performed too early in the infection, it will be falsely negative because there may not yet be enough antibodies against HIV to trigger a positive result.

A middle-aged man who is homeless reports to the local public health clinic complaining of a painless and shallow ulcer on the penile shaft for the past 2 weeks. He is sexually active and had unprotected intercourse with two male partners over the past few months. The patient is tested for HIV, syphilis, gonorrhea, hepatitis B, and herpes types 1 and 2. The syphilis and HIV tests are both positive. The gonorrhea, hepatitis B, and herpes tests are negative. The nurse practitioner is aware of the nationally notifiable infectious conditions. Which of the following is true regarding reporting of any of these sexually transmitted infections? A) Health care providers must obtain the patient's permission before reporting the positive HIV and syphilis test results to the local public health department B) The nurse practitioner should obtain the patient's and sexual partner's permission before reporting the positive test results to the local health department C) Health care providers are mandated by law to report certain types of diseases to the local health department even if the patient does not give permission D) The nurse practitioner should consult with the supervising physician about this issue

C) Health care providers are mandated by law to report certain types of diseases to the local health department even if the patient does not give permission Physicians and laboratories are legally mandated to report certain types of diseases. Sexually transmitted diseases, HIV infection/AIDS, gonorrhea, and syphilis must be reported to the local health department even if the patient does not give permission. Partner tracing and notification are done by the local health department. The Centers for Disease Control and Prevention (CDC) website contains a list of nationally reportable diseases. Other diseases that are on the CDC 2017 list of reportable diseases (i.e., diseases that must be reported) are tuberculosis; diphtheria; hepatitis A, B, and C; measles; mumps; pertussis; Lyme disease; Rocky Mountain spotted fever; and many others.

All of the following are true about strawberry hemangiomas found in infants except: A) Most will involute spontaneously by the age of 18 to 24 months B) Watchful waiting is the most useful strategy C) Hemangiomas should be treated with laser therapy if they have not resolved by the age of 12 months D) Strawberry hemangiomas are benign

C) Hemangiomas should be treated with laser therapy if they have not resolved by the age of 12 months True strawberry hemangiomas will eventually resolve by the time the child goes to kindergarten. Most will reduce or disappear in the first 2 years. Laser treatment is rarely needed.

A 19-year-old woman has recently been diagnosed with acute hepatitis B. She is sexually active and is monogamous. She reports that her partner uses condoms inconsistently. What would you recommend for her male sexual partner who was also tested for hepatitis with the following results: HBsAg (−), anti-HBs (−), anti-HCV (−), anti-HAV (+)? A) Hepatitis B vaccination B) Hepatitis B immunoglobulin C) Hepatitis B vaccination and hepatitis B immunoglobulin D) No vaccination is needed at this time

C) Hepatitis B vaccination and hepatitis B immunoglobulin Hepatitis B surface antigen (HBsAg) is a marker of infectivity. If positive, it indicates either an acute or a chronic hepatitis B infection. Antibody to hepatitis B surface antigen (anti-HBs) is a marker of immunity. Antibody to hepatitis B core antigen (anti-HBc) is a marker of acute, chronic, or resolved hepatitis B virus (HBV) infection; it may be used in prevaccination testing to determine previous exposure to HBV. The hepatitis B panel results for the individual in this question (negative HBsAg, anti-HBc, and anti-HBs) indicates the partner is susceptible (not immune), has not been infected, and is still at risk of future infection—and thus needs vaccine. Interpretation of the negative hepatitis C anti-HCV screening test indicates that the partner is not infected. Hepatitis B immunoglobulin contains antibodies that provide "instant" immunity against hepatitis B, but its action lasts for several days, only. It is not a vaccine. It is given to infants and others who are at high risk of becoming infected and are not immune. The hepatitis B vaccine stimulates the body to make its own antibodies, which are permanent. A total of three doses are needed to gain full immunity against hepatitis B.

The mother of a 13-year-old boy with Down syndrome is in the family nurse practitioner's office and wants to schedule a sports physical for her son. She reports that he wants to join the football team at his school. You would tell the mother that her son: A) Can play a regular football game as long as he wears maximum protective football gear B) Cannot play some contact sports because of an increased risk of cervical spine injury C) Can play certain contact sports after he has been checked for cervical instability D) None of the above

C) Her son can participate in some sports after he has been checked for cervical instability Atlantoaxial instability (AAI) denotes increased mobility at the articulation of the first and second cervical vertebrae (atlantoaxial joint). The American Academy of Pediatrics issued a position statement in 1984 on AAI and Down syndrome (DS): All children with DS who wish to participate in sports should have cervical spine x-rays. Repeated x-rays are not indicated for children with DS who have had a previously normal neck x-ray. Persons with DS who have no evidence of AAI may participate in all sports.

A new patient who is a 40-year-old female postal worker is being evaluated for complaints of a new-onset erythematous rash on both cheeks and the bridge of the nose, accompanied by fatigue. She reports a history of Hashimoto's thyroiditis and is currently being treated with Synthroid 1.25 mg daily. Which of the following conditions is most likely? A) Atopic dermatitis B) Thyroid disease C) Lupus erythematosus D) Rosacea

C) Lupus erythematosus Classic symptoms of lupus erythematosus are butterfly rash across both cheeks and the bridge of the nose, and fatigue. Risk factors also include being female and 40 years old.

All of the following infections are reportable diseases except: A) Lyme disease B) Gonorrhea C) Nongonococcal urethritis D) Syphilis

C) Nongonococcal urethritis A number of communicable diseases are required to be reported to the state public health department. They include Lyme disease, gonorrhea, and syphilis. Statistics of reportable diseases are kept in each state, and a subset is then reportable to the Centers for Disease Control and Prevention (CDC). The most common bacterial cause of nongonococcal urethritis (NGU) is Chlamydia trachomatis, but it may also be caused by Ureaplasma urealyticum, Haemophilus vaginalis, or Mycoplasma genitalium. Viral causes include herpes simplex and adenovirus; rarely, a Trichomonas vaginalis (parasite) may cause NGU. It can also be caused by a mechanical injury such as a catheter or cystoscopy.

A neighbor's 14-year-old son, who is active in basketball, complains of pain and swelling on both knees. On physical exam, there is tenderness over the tibial tuberosity of both knees. Which of the following is most likely? A) Chondromalacia patella B) Left knee sprain C) Osgood-Schlatter disease D) Tear of the medial ligament

C) Osgood-Schlatter disease Osgood-Schlatter disease is characterized by bilateral pain over the tibial tuberosity upon palpation, along with knee pain and edema with exercise.

A 15-year-old basketball player who is 6 ft tall is seen for complaints of painful lumps on his knees. Upon inspection, the nurse practitioner notes a bonelike growth on the upper tibia midline below the kneecap on both knees. The patient has full range of motion with no joint tenderness, redness, or swelling. Which of the following conditions is best described? A) Osteosarcoma of the tibia B) Juvenile rheumatoid arthritis C) Osgood-Schlatter disease D) Paget's disease of the bone

C) Osgood-Schlatter disease Osgood-Schlatter disease is characterized by pain over the tibial tuberosity with palpation of a bony mass over the anterior tubercle of one or both knees. Exercise worsens the pain.

A 13-year-old boy wants to be treated for his acne. He has a large number of closed and open comedones on his face. The patient has been treating himself with over-the-counter benzoyl peroxide and topical salicylic acid products. Which of the following would be recommended next? A) Isotretinoin (Accutane) B) Tetracycline C) Retin-A 0.25% gel D) Careful face washing with medicated soap at bedtime

C) Retin-A 0.25% gel Topical agents are the first-line treatment for acne vulgaris. Retin-A 0.25% gel would be the next step. Oral preparations (tetracycline) would then be offered, and Accutane would be the final step.

A new patient who recently visited a relative in North Carolina complains of an onset of fever and red rashes that started 2 days ago. The rash first appeared on the wrist and the ankles and included the palms of the hands. The patient reports that it is spreading toward his trunk. The patient's eyes are not injected and no enlarged nodes are palpated on his neck. There is no desquamation of the skin. Which of the following is most likely? A) Kawasaki's disease B) Meningococcemia C) Rocky Mountain spotted fever D) Measles

C) Rocky Mountain spotted fever Given the location in the East, Rocky Mountain spotted fever is most likely the infection causing symptoms of fever and a rash on ankles and wrists, moving to the palms of the hands and the trunk. Rocky Mountain spotted fever is caused by the bacterium Rickettsia rickettsii, which is transmitted by a bite from an infected tick.

In most states, patients younger than age 18 years may consent to health care without parental or legal guardian consent in all except which of the following cases? A) Contraception B) Pregnancy C) School physical exams D) Sexually transmitted disease (STD) evaluation and treatment

C) School physical exams Any student younger than 18 years of age must have parental permission to have a school physical exam done.

A girl whose breasts form a secondary mound is at which Tanner stage? A) Tanner stage II B) Tanner stage III C) Tanner stage IV D) Tanner stage V

C) Tanner stage IV The breast bud stage is Tanner stage II. In Tanner stage IV, the areola and papilla form a secondary mound. Tanner stages for breast development are I, prepuberty; II, breast bud; III, breast and areola one mound; IV, breast and areola secondary mound; and V, adult pattern.

A 10-year-old boy complains of sudden onset of scrotal pain upon awakening that morning. He is also complaining of severe nausea and vomiting. During the physical examination, the nurse practitioner finds a tender, warm, and swollen left scrotum. The cremasteric reflex is negative and the urine dipstick is negative for leukocytes, nitrites, and blood. The most likely diagnosis is: A) Acute epididymitis B) Severe Salmonella infection C) Testicular torsion D) Acute orchitis

C) Testicular torsion Signs and symptoms of testicular torsion include sudden onset of unilateral scrotal pain, nausea, vomiting, and abdominal pain. Acute epididymitis causes fever, chills, nausea, and unilateral pain and is most commonly seen in sexually active men. Unilateral scrotal pain does not occur with Salmonella infection. Acute orchitis is often based on having a recent mumps infection (parotitis) with testicular edema.

A 19-year-old student who is on a prescription of combined oral contraceptive pills is being seen for an annual gynecological exam in the college health center. The nurse practitioner has obtained the Pap smear and is about to perform the bimanual exam. She gently removes the plastic speculum from the vagina. While the NP is performing the bimanual vaginal exam, the patient complains of slight discomfort during deep palpation of the ovaries. Which of the following is a true statement? A) The uterus and the ovaries are both very sensitive to any type of palpation B) The fallopian tubes and ovaries are not sensitive to light or deep palpation C) The ovaries are sensitive to deep palpation but they should not be painful D) The uterus and the ovaries are not important organs of reproduction

C) The ovaries are sensitive to deep palpation but they should not be painful The ovaries are usually slightly sensitive to deep palpation, but they should not be painful. Unilateral adnexal pain accompanied by cervical motion tenderness and purulent endocervical discharge is suggestive of pelvic inflammatory disease (PID)

A sexually active 22-year-old man is asking to be screened for hepatitis B because his new girlfriend has recently been diagnosed with hepatitis B infection. His lab results are the following: anti-HBV is negative, HBsAg is positive, and HBeAg is negative. Which of the following is true? A) The patient is immune to the hepatitis B virus B) The patient is not infected with hepatitis B virus C) The patient needs hepatitis B vaccine and hepatitis B immunoglobulin D) The patient needs only hepatitis B immunoglobulin

C) The patient needs hepatitis B vaccine and hepatitis B immunoglobulin Because he is HBsAg positive, and anti-HBV and HBeAg negative, he needs hepatitis B immunoglobulin and hepatitis B vaccine.

You suspect an enterobiasis infection in a 6-year-old girl. Which of the following tests would you recommend? A) Stool culture and sensitivity B) Stool for ova and parasites C) The scotch tape test D) Hemoccult test

C) The scotch tape test Enterobiasis infection (pinworms) is caused by small worms that infect the intestines. Symptoms include itching around the anus, which is usually worse at night. The scotch tape test is done by applying the scotch tape on the anal area in the morning; the worms commonly come out at night and will stick to the tape, which is used for diagnosis.

A 16-year-old female patient is being treated for her first urinary tract infection. She had an allergic reaction with hives after taking sulfa as a child. Which of the following antibiotics would be contraindicated? A) Cephalexin (Keflex) B) Ampicillin (Amoxil) C) Trimethoprim-sulfamethoxazole (Bactrim) D) Nitrofurantoin crystals (Macrobid)

C) Trimethoprim-sulfamethoxazole (Bactrim) With the allergic history to a sulfa drug, it would be safest to avoid Bactrim.

A 22-year-old sexually active woman is complaining of amenorrhea and new-onset bloody vaginal spotting. On examination, her left adnexa is tender and cervical motion tenderness is positive. Which test should the nurse practitioner order initially? A) Flat plate of the abdomen B) Complete blood count (CBC) with white cell differential C) Urine pregnancy test D) Pelvic ultrasound

C) Urine pregnancy test The patient's history of amenorrhea and new onset of bloody vaginal spotting combined with positive physical findings of left adnexal tenderness and cervical motion tenderness are highly suggestive of an ectopic pregnancy rather than pelvic inflammatory disease (PID). Refer this patient to the emergency department if ectopic pregnancy is suspected. The presence of amenorrhea should be treated as a pregnancy until proven otherwise.

In small children with acquired immunodeficiency syndrome (AIDS), which of the following vaccines is contraindicated? A) Diphtheria and tetanus (Td) B) Hepatitis B and mumps C) Varicella D) Td and oral polio

C) Varicella The data regarding efficacy of the varicella vaccine are insufficient; therefore, varicella vaccine is contraindicated in HIV-infected individuals.

Which of the following is the best method to diagnose a vaginal trichomonas infection? A) Whiff test B) Potassium hydroxide (KOH) test C) Wet smear with microscopy D) Culture

C) Wet smear with microscopy The best method of diagnosing vaginal trichomonas infection in primary care is by wet smear with microscopy. Trichomonas is a protozoan; it is a unicellular organism with flagella. Place a small amount of vaginal discharge on a glass slide, add a few drops of normal saline, and place the cover slip on top. Do not use potassium hydroxide (KOH) as it will rupture the organisms. Trichomonas can be visualized by microscopy.

When a domestic dog is suspected of being infected with the rabies virus, it can either be killed for a brain biopsy or it can be quarantined. What is the minimum number of days a dog suspected of rabies must be quarantined? A) 4 weeks B) 21 days C) 14 days D) 10 days

D) 10 days The minimum number of days to quarantine an animal suspected of rabies is 10 days. If the animal is healthy and has no symptoms of rabies at 10 days, it is not infected with the rabies virus and can be returned to the owner.

The earliest age at which the MMR vaccine can be administered is: A) 4 months B) 6 months C) 8 months D) 12 months

D) 12 months The earliest age at which MMR vaccination should be given is 12 months. Giving the vaccine any earlier may be less effective because the infant still has antibodies from the mother, which may interfere with the production of the antibodies stimulated by the vaccine.

The nurse practitioner does not need to obtain parental consent from all of the following patients except: A) A 17-year-old who wants to be treated for a sexually transmitted infection B) A 12-year-old who wants a serum pregnancy test C) A 15-year-old who wants birth control pills D) A 14-year-old who wants to be treated for dysmenorrhea

D) A 14-year-old who wants to be treated for dysmenorrhea Treatment for teenagers may be done without parental consent for sexually transmitted infections, pregnancy testing, and contraception counseling and treatment. Parental consent is required for any type of physical exam or for other problems that require more invasive testing.

You would associate a positive iliopsoas muscle test result with which of the following conditions? A) Left cerebral vascular accident B) Urinary tract infection C) Heel fractures D) Acute appendicitis

D) Acute appendicitis A positive iliopsoas muscle test may be seen with acute appendicitis. The right iliopsoas muscle lies under the appendix, This test is performed by asking the patient to actively flex the thigh at the hip. A "positive psoas sign" is noted when the patient exhibits pain in the right lower quadrant due to the inflamed tissue.

A 35-year-old sexually active man presents with a 1-week history of fever and pain over the left scrotum. It is accompanied by frequency and dysuria. The scrotum is edematous and tender to touch. He denies flank pain, nausea, and vomiting. He reports that the pain is lessened when he uses scrotal-support briefs. The urinalysis shows 2+ blood and a large number of leukocytes. What is the most likely diagnosis? A) Acute urinary tract infection B) Acute pyelonephritis C) Acute orchitis D) Acute epididymitis

D) Acute epididymitis Acute epididymitis is the infection presented here. Scrotal edema and pain with palpation do not occur in urinary tract infection or pyelonephritis. Acute orchitis symptoms include testicular pain and edema, are usually associated with the mumps, but do not have frequency and dysuria.

All of the following clinical signs and symptoms are seen early in testicular torsion except: A) Nausea and vomiting B) Absence of the cremasteric reflex C) Affected testicle is elevated compared with the normal testicle D) Affected testicle is swollen and feels cold to touch

D) Affected testicle is swollen and feels cold to the touch The affected testicle will be swollen, but it will feel very warm to the touch. A cold testicle is abnormal and is indicative of gangrene (after 24 hours).

When a patient is suspected of having acute pancreatitis, initial testing should include all of the following except: A) Electrolyte panel B) Serum amylase level C) Serum lipase level D) Barium swallow

D) Barium swallow When a patient has acute abdominal pain, initial labs performed are serum electrolytes, amylase, and lipase. Barium swallow would not be performed initially.

A 15-year-old boy has just moved into the community and is staying in a foster home temporarily. There is no record of his immunizations. His foster mother wants him to be checked before he enters the local high school. Which of the following immunizations does this patient need? A) Meningococcal vaccine B) Measles-mumps-rubella (MMR) vaccine C) Tdap vaccine D) All of the above

D) All of the above This 15-year-old teenage patient will follow the Centers for Disease Control and Prevention (CDC) "catch-up" schedule (CDC, 2017) and needs the meningococcal vaccine, MMR vaccine, and Tdap vaccine. In addition to these three vaccines, this patient needs the hepatitis B vaccine, human papillomavirus (HPV) vaccine, and varicella vaccine (if no history of chickenpox).

You are performing a pelvic exam on a 25-year-old sexually active woman. You palpate a tender and warm cystic mass on the lower edge of the left labia majora, which is red. The most likely diagnosis is: A) Skene's gland cyst B) Cystocele C) Lymphogranuloma venereum D) Bartholin's gland abscess

D) Bartholin's gland abscess Bartholin's glands are located in the base of the labia minora at about the 4 o'clock and 8 o'clock positions. Their function is to provide moisture for the vestibule. They are small (about pea sized) unless they become clogged or infected. If this occurs, an abscess may form and glands will enlarge and become painful.

Systemic lupus erythematosus (SLE) is more common among patients from all of the following ethnic and racial backgrounds except: A) African American B) Asian C) Hispanic D) Caucasian

D) Caucasian Lupus is more common among women of African American, Asian, Hispanic, and Native American backgrounds than in Caucasian women.

A 14-year-old boy is brought in by his mother who reports that her son has been complaining for several months of recurrent bloating, stomach upset, and occasional loose stools. She reports that he has difficulty gaining weight and is short for his age. She has noticed that his symptoms are worse after eating large amounts of crackers, cookies, and breads. She denies seeing blood in the boy's stool. Which of the following conditions is most likely? A) Amebiasis B) Malabsorption C) Crohn's colitis D) Celiac disease

D) Celiac disease Celiac disease is also known as celiac sprue. Patients should avoid foods containing gluten, which causes malabsorption (diarrhea, gas, bloating, and abdominal pain). Foods to avoid are wheat, rye, and barley. Oats do not damage the mucosa in celiac disease. Antigliadin IgA and IgG are elevated in almost all patients (90%).

All of the following are infections that affect mostly the labia and vagina except: A) Bacterial vaginosis B) Candidiasis C) Trichomoniasis D) Chlamydia trachomatis

D) Chlamydia trachomatis Infections that commonly affect the labia and vagina include bacterial vaginosis, candidiasis, and trichomoniasis. Chlamydia trachomatis commonly affects the cervix, endometrial lining, fallopian tubes, and pelvic cavity

A male 16-year-old with a recent history of a cat bite is brought to the walk-in clinic by his mother. The bite occurred about 2 hours before the visit. The nurse practitioner evaluates the wound and notes two small puncture wounds. There is no redness or purulent discharge. The mother reports that the teenager received a tetanus booster when he was 12 years old. Which of the following is the correct action to take? A) Clean the wound with soap and water and apply a topical antibiotic and a bandage B) Because the wound is clean and does not appear infected, there is no need for antibiotics C) Give the patient a tetanus booster using the Tdap form of the vaccine D) Clean the wound with soap and water and prescribe amoxicillin-clavulanate (Augmentin) 500 mg PO BID × 10 days

D) Clean the wound with soap and water and prescribe amoxicillin-clavulanate (Augmentin) 500 mg PO BID × 10 days Cat bites are more likely to become infected than dog bites, and this bite is located on an extremity. These facts justify the prescription of Augmentin 500 mg PO BID × 10 days for the patient. Because the last tetanus booster was given at age 13 years, it has only been 4 years since the patient's last tetanus booster. This incident falls within the 5-year time period in which the previous booster would offer protection; hence there is no need for a tetanus booster at this visit.

A 5-year old boy is in your office for a school physical. The mother denies a history of chickenpox infection. Which of the following immunizations is indicated at this visit? A) Tdap, IPV, MMR B) DTaP, Hib, PCV, IPV C) MMR, hepatitis B, varicella D) DTaP, IPV, MMR, varicella

D) DTaP, IPV, MMR, varicella The best clue is that the mother denies a history of chickenpox infection; therefore, this child requires the varicella vaccine. Only two answer options include varicella. The child does not need hepatitis B vaccination.

A 21-year-old woman complains of left-sided pelvic pain accompanied by dyspareunia. During the gynecological exam, the nurse practitioner notices green cervical discharge. The patient mentions a new onset of a painful and swollen left knee and denies a history of trauma. This best describes: A) Septic arthritis B) Reiter's syndrome C) Chondromalacia of the patella D) Disseminated gonorrheal infection

D) Disseminated gonorrheal infection Symptoms of pelvic inflammatory disease (PID) with painful, swollen joints of extremities indicate disseminated gonorrheal infection. Untreated disseminated gonorrhea can lead to septic arthritis. Symptoms may be mild, ranging from slight joint pain and no fever to severe joint pain with high fever. PID symptoms do not occur with septic arthritis, Reiter's syndrome, or chondromalacia of the patella.

All of the following abnormal lab results may be seen in patients with infectious mononucleosis except: A) Lymphocytosis B) Positive EBV (Epstein-Barr virus) titers for IgM and IgG C) Abnormal liver function tests D) Elevated creatinine level

D) Elevated creatinine level Infectious mononucleosis is caused by Epstein-Barr virus (EBV). Symptoms consist of lymphadenopathy, fever, hepatosplenomegaly, malaise, and abdominal discomfort in adolescents and young adults. Host immune response to the viral infection includes CD8+ T lymphocytes with suppressor and cytotoxic functions, the characteristic atypical lymphocytes found in the peripheral blood. EBV antibodies may be ordered when symptoms suggest mononucleosis, but a mono test is negative. A positive test for IgM antibodies is most likely a current, or a very recent, EBV infection. With a positive IgG concentration, it is highly likely that the patient recently had an EBV infection. EBV infection does not infect the kidneys; therefore, the serum creatinine is not affected. But EBV can infect the brain/nerves, bone marrow, liver, and other organs. It is also associated with certain cancers.

You would recommend the pneumococcal vaccine (Pneumovax) to patients with all of the following conditions except: A) Sickle cell anemia B) Splenectomy C) HIV infection D) G6PD-deficiency anemia

D) G6PD-deficiency anemia Pneumococcal vaccine is not indicated for glucose-6-phosphate dehydrogenase (G6PD)-deficiency anemia. There two types of pneumonia vaccine for adults: PPSV23 (Pneumovax 23) and PCV13 (Prevnar 13). Pneumococcal vaccine is recommended for individuals beginning at age 65 years, but the two types should not be given together. The Centers for Disease Control and Prevention (CDC) recommends administering Prevnar 13 first, then waiting at least 1 year (12 months) and giving the Pneumovax. The immunogenic response is better using this method. Prevnar 13 is recommended for all infants and children younger than 2 years of age, all adults aged 65 years or older, and people (2 to 64 years old) with certain medical conditions that increase risk of pneumococcal disease, such as functional or anatomic asplenia (sickle cell), HIV infection, chronic renal failure, leukemia, heart failure, cyanotic congenital heart disease, chronic lung disease (asthma, chronic obstructive pulmonary disease [COPD]), diabetes, and others.

All of the following are considered risk factors for urinary tract infections in women except: A) Diabetes mellitus B) Diaphragms and spermicide use C) Pregnancy D) Intrauterine device

D) Intrauterine device An intrauterine device (IUD) is not a risk factor for urinary tract infections (UTIs). In addition to causing increased urine glucose, diabetes may increase the risk of UTIs through mechanisms that include impaired immune cell delivery, inefficient white blood cells, and inhibition of bladder contractions that allow urine to remain stagnant in the bladder. Diaphragms are associated with an increased risk of UTIs. Urinating before inserting the diaphragm and also after intercourse may reduce this risk. Hormonal and mechanical changes increase the risk of urinary stasis and vesicoureteral reflux. These changes, along with an already short urethra (approximately 3-4 cm in females) and difficulty with hygiene due to a distended pregnant belly, increase the frequency of UTIs in pregnant women. Indeed, UTIs are among the most common bacterial infections during pregnancy.

Which of the following physical exam findings is most specific for systemic lupus erythematosus (SLE)? A) Swollen and painful joint involvement B) Fatigue and myalgia C) Stiffness and swelling of multiple joints D) Malar rash

D) Malar rash A malar rash is the butterfly-shaped rash on the middle of the face that is caused by a type of photosensitivity reaction. It is associated with systemic lupus erythematosus (SLE). The other answer options are found with diseases such as rheumatoid arthritis, polymyalgia rheumatica, and others.

Patients with Down syndrome are at higher risk for all of the following except: A) Atlantoaxial instability B) Congenital heart disease C) Early onset of Alzheimer's disease D) Melanoma

D) Melanoma Patients with Down syndrome are at higher risk for atlantoaxial instability, congenital heart defects, and early onset of Alzheimer's disease. Children with Down syndrome who participate in certain sports activities must be carefully examined for an unstable neck and heart disease prior to participation to prevent injury.

The differential diagnosis for genital ulceration includes all of the following except: A) Syphilis B) Genital herpes C) Chancroid D) Molluscum contagiosum

D) Molluscum contagiosum Genital ulcers may occur with syphilis, genital herpes, and chancroid. Molluscum contagiosum is a viral skin infection. In adults, molluscum contagiosum appears on the face, neck, armpits, arms, and hands. Other common places include the genitals, abdomen, and inner thigh. Lesions often begin as small, firm, domeshaped growths; have a surface that feels smooth, waxy, or pearly; are flesh-colored or pink; have a dimple in the center (may be filled with a thick, white substance that is cheesy or waxy); and are painless but itch. Scratching or picking can spread the virus.

All of the following clinical findings are major or minor criteria for pelvic inflammatory disease (PID). Which of the following is classified as a minor criterion that is not required for diagnosis of PID? A) Cervical motion tenderness B) Adnexal tenderness C) Uterine tenderness D) Oral temperature >101°F (>38°C)

D) Oral temperature >101°F (>38°C) Pelvic inflammatory disease (PID) is a clinical diagnosis. The presence of at least one of the major criteria (cervical motion tenderness, adnexal tenderness, uterine tenderness) when combined with the history is highly suggestive of PID. Minor criteria are not necessary, but they help to support the diagnosis of PID (oral temperature >101°F [>38°C]), mucopurulent cervical or vaginal discharge, elevated sedimentation rate, elevated C-reactive protein, large amount of WBCs on saline microscopy of the vaginal fluid, or laboratory documentation of cervical infection with Neisseria gonorrhoeae or Chlamydia trachomatis).

Women with a history of pelvic inflammatory disease (PID) have an increased risk for all of the following complications except: A) Ectopic pregnancy B) Scarring of the fallopian tubes C) Infertility D) Ovarian cysts

D) Ovarian cysts Women with a history of pelvic inflammatory disease (PID) have a higher risk of ectopic pregnancy, scarring of the fallopian tubes, and infertility due to the scarring of the fallopian tube(s).

An 18-year-old college student, who is sexually active, wants to start taking birth control pills. The nurse practitioner at the college health clinic explains the risks and benefits of combined oral contraceptives to the patient. All of the following are appropriate laboratory tests for this patient except: A) Chlamydia test B) Gonorrhea test C) HIV test D) Pap smear

D) Pap smear The U.S. Preventive Services Task Force (USPSTF; 2012) does not recommend Pap smears (cytology) in women younger than 21 years of age because cervical cancer is rare before this age. A history of sexual intercourse or multiple sex partners is not considered to be an indication for a Pap smear in this age group. There is evidence that screening earlier than age 21 years, regardless of sexual history, would lead to more harm than benefit (USPSTF, 2012). Instead, testing for sexually transmitted disease/infection is the best choice for this age group. The USPSTF recommends screening for cervical cancer in women aged 21 to 65 years with cytology (Pap smear) every 3 years. Another option for women aged 30 to 65 years who want to lengthen the screening interval is a combination of cytology with HPV testing (cotesting) every 5 years.

Which of the following conditions is associated with a positive Auspitz sign? A) Contact dermatitis B) Seborrheic dermatitis C) Systemic lupus erythematosus D) Psoriasis

D) Psoriasis The Auspitz sign is simply bleeding that occurs after psoriasis scales have been removed. It occurs because the capillaries run very close to the surface of the skin under a psoriasis lesion, and removing the scale essentially pulls the tops off the capillaries, causing bleeding. Auspitz sign is also found in other scaling disorders such as actinic keratoses.

A woman who has recently been diagnosed with lupus complains that her hands and feet always feel cold even in the summertime. Sometimes her fingertips become numb and turn a blue color. The fingertips eventually turn dark red in color. Which of the following is most likely? A) Chronic arterial insufficiency B) This is a normal reaction when one feels very cold C) Peripheral vascular disease D) Raynaud's phenomenon

D) Raynaud's phenomenon Raynaud's phenomenon occurs from vasospasms of the blood vessels, leading to decreased blood supply to the hands and feet, which causes bluish discoloration, with fingertips turning a dark-red color if severe. Stress and cold weather are classic triggers for this syndrome.

The mother of an 11-year-old boy with sickle cell anemia calls on the phone because her son woke up with a painful penile erection that will not go away. The nurse practitioner's most appropriate intervention is: A) Insert a Foley catheter and measure the child's intake and output for the next 24 hours B) Insert a Foley catheter to obtain a specimen for a urinalysis and urine for C&S (culture and sensitivity) C) Recommend an increase in the child's fluid intake D) Recommend immediate referral to the emergency department

D) Recommend immediate referral to the emergency department Priapism (painful penile erection not related to sexual activity) is a true urological emergency that may lead to permanent erectile dysfunction and penile necrosis if not treated appropriately. It can be associated with a number of medical conditions (sickle cell anemia, leukemia, or spinal cord injury) and/or some pharmacological agents.

Your patient is being worked up for microscopic hematuria. All of the following are differential diagnoses of microscopic hematuria except: A) Kidney stones B) Bladder cancer C) Acute pyelonephritis D) Renal artery stenosis

D) Renal artery stenosis Renal artery stenosis refers to narrowing of the kidney arteries. It is commonly noted in individuals older than 50 years of age and is associated with atherosclerosis and hypertension. Hematuria is not associated with renal artery stenosis. Evidence of blood in the urine can be seen with kidney stones, bladder cancer, and acute pyelonephritis.

An 18-year-old waitress is diagnosed with pelvic inflammatory disease (PID). The cervical Gen-Probe result is positive for Neisseria gonorrhoeae and negative for Chlamydia trachomatis. All of the following statements are true regarding the management of this patient except: A) This patient should be treated for chlamydia even though the Gen-Probe for chlamydia is negative B) Ceftriaxone 250 mg IM and doxycycline 100 mg PO BID × 14 days are appropriate treatment for this patient C) Advise the patient to return to the clinic for a follow-up visit within 3 days after treatment D) Repeat the Gen-Probe test for Chlamydia trachomatis to ensure that the previous test was not a false-negative result

D) Repeat the Gen-Probe test for Chlamydia trachomatis to ensure that the previous test was not a false-negative result Treatment for both gonorrhea and chlamydia is recommended for the diagnosis of pelvic inflammatory disease (PID), regardless of whether the chlamydia test (or gonorrhea test) was negative. Repeating the Gen-Probe test (or another nucleic acid amplification test [NAAT]) for chlamydia is not recommended. Pelvic inflammatory disease (PID) is a clinical diagnosis. The Centers for Disease Control and Prevention (CDC) states that an internal vaginal swab specimen is equivalent to a cervical specimen when using the newer NAATs. Self-collected vaginal swabs are equivalent in sensitivity/specificity to those collected by a clinician. Treatment should be initiated as soon as the presumptive diagnosis has been made because prevention of long-term sequelae is dependent on early administration of appropriate antibiotics. All regimens used to treat PID should also be effective against N. gonorrhoeae and C. trachomatis because negative endocervical screening for these organisms does not rule out upper reproductive tract infection.

A 13-year-old girl is brought by her mother to the health clinic because the girl is complaining of vaginal discharge and pain. The mother tells the nurse practitioner that her daughter is not sexually active yet. The mother is divorced, lives with her boyfriend, and works full time. During the exam, the nurse practitioner notes that the vaginal introitus is red, with tears and a torn hymen. The cervix is covered with green discharge. The nurse practitioner suspects that the child has been sexually abused by the mother's boyfriend. What is the best action for the nurse practitioner to take? A) Ask the mother questions about her boyfriend's behaviors B) Advise the mother to watch how her boyfriend interacts with her daughter and to call within 1 week to discuss his behavior with her C) Advise the mother that you suspect that her daughter has been sexually abused D) Report the child abuse to the department of child protective services

D) Report the child abuse to the department of child protective services There are several "helping" professions (nurses, teachers, mental health) that are required to report suspected or actual child/elderly abuse to authorities. The nurse practitioner is legally required to report the case to child protective services (CPS). If the child is in danger, CPS may ask for a court order to take the child away for protection until the investigation is completed. Talking about the boyfriend's behavior will not be effective and may put the child and/or mother in danger if the boyfriend suspects that he is being watched.

All of the following viruses can infect the fetus and/or the mother during pregnancy. Which of the following viruses is not associated with serious sequelae? A) Human parvovirus B19 B) Varicella C) Rubella D) Rhinovirus

D) Rhinovirus Infection with rhinovirus causes the common cold and is not associated with serious sequelae in pregnant women or their fetuses. Infection during pregnancy with human parvovirus B19 can cause severe fetal anemia (hemolysis), fetal hydrops (hydrops fetalis), fetal death, and spontaneous abortions. Most women with B19 infection in pregnancy do not suffer serious sequelae. Congenital rubella infection in early pregnancy causes severe birth defects, fetal demise, miscarriages, and stillbirths. Congenital varicella infection is extremely rare but affects the skin, extremities, and brain, and also causes low birth weight.

A 4-week-old boy is seen in the family practice clinic for a complaint of forceful vomiting that occurs immediately after feeding. The vomitus is composed of infant formula and is not bilious. The infant is bottle-fed with infant formula that was recommended by the pediatrician. The mother reports that the infant seems hungry and sucks on the bottle without any problems. His birth weight was 7 lbs, 5 oz (3.4 kg). The current weight is 7 lbs (3.2 kg). Which of the following clinical findings is an important clue regarding the possible cause of the infant's vomiting? A) Irritable and crying infant B) Sunken anterior fontanel and dry lips C) Positive rooting reflex D) Round olive-like mass located in the right upper quadrant of the abdomen

D) Round olive-like mass located in the right upper quadrant of the abdomen The stem is asking about the "possible cause of the infant's vomiting" (it is not asking about symptoms). Projectile or forceful vomiting after feeding (postprandial vomiting) is a classic symptom of infantile hypertrophic pyloric stenosis. A hypertrophied pylorus is a pathognomonic finding of the disease. An ultrasound of the pylorus is the imaging study of choice. The other signs and symptoms are dehydration (sunken anterior fontanel, dry lips, weight loss), irritability, and crying (usually due to hunger). A positive rooting reflex is a normal finding in a 4-week-old infant.

A preschool girl who is homeschooled is brought by her mother to the walk-in clinic because of acute onset of fever, runny nose, cough, sore throat, and red eyes with a morbilliform rash. The mother reports that her daughter has never been immunized. The family recently returned from a vacation. Which of the following conditions is the most likely? A) Rubella B) Varicella C) Fifth disease D) Rubeola

D) Rubeola Rubeola, also known as measles, is caused the rubeola virus. It is very contagious and is transmitted via droplets like the common cold. Koplik's spots (tiny white spots in buccal mucosa) are present during the prodromal period. The blotchy pink rash is also known as a morbilliform rash. Treatment is symptomatic and most people recover in 2 to 3 weeks. Measles outbreaks caused by poor MMR (measles, mumps, rubella) vaccination rates have been reported in some areas of the United States. Do not confuse rubeola with rubella (German measles).

A new patient is complaining of severe pruritus that is worse at night. Several family members also have the same symptoms. Upon examination, areas of excoriated papules are noted on some of the interdigital webs of both hands and on the axillae. This finding is most consistent with: A) Contact dermatitis B) Impetigo C) Larva migrans D) Scabies

D) Scabies Scabies is a parasitic disease (infestation) of the skin caused by the human itch mite Sarcoptes scabiei. The rash is generally characterized as red, raised excoriated papules. The scabies mite is generally transmitted from one person to another by direct contact with the skin of the infested person and can also be acquired by wearing an infested person's clothing (fomites), such as sweaters, coats, or scarves. Following the incubation period, the infested person will complain of pruritus (itching), which intensifies at bedtime under the warmth of the blankets. Common sites of infection are the webs of fingers, wrists, flexors of the arms, the axillae, lower abdomen, genitalia, buttocks, and feet.

The cremasteric reflex is elicited by: A) Asking the patient to open his or her mouth and touching the back of the pharynx with a tongue blade B) Hitting the biceps tendon briskly with a reflex hammer and watching the lower arm for movement C) Hitting the patellar tendon briskly with a reflex hammer and watching the lower leg for movement D) Stroking the inner thigh of a male patient and watching the testicle on the ipsilateral side rise up toward the body

D) Stroking the inner thigh of a male patient and watching the testicle on the ipsilateral side rise up toward the body The cremasteric reflex is elicited by stroking the inner thigh (proximal to distal) with a blunt instrument such as a handle of the reflex hammer. The testicle and scrotum should rise on the stroked (ipsilateral) side.

A 45-year-old gardener is seen as a walk-in patient in a private clinic. He reports stepping on a nail that morning. His last tetanus vaccine was 7 years ago. Which of the following vaccines is recommended? A) DTaP B) DT C) Td D) Tdap

D) Tdap The Centers for Disease Control and Prevention (CDC) recommends that one of the tetanus boosters be replaced with the Tdap (once in a lifetime). Thereafter, the Td form of the vaccine is indicated every 10 years. The DTaP (diphtheria-tetanus-acellular pertussis) and DT (diphtheria-tetanus) forms of the tetanus vaccine are not given after the age of 7 years. Puncture wounds are at higher risk for tetanus because Clostridium tetani bacteria are anaerobes (deep puncture wounds are not exposed to air compared with superficial wounds).

A 25-year-old man who was involved in a car accident is brought to the local emergency department. He reports wearing a seat belt and was the driver of the vehicle. The patient is complaining of pain on his right lower leg. The right foot is abducted and the ankle is swollen and bruised. There is a small laceration on the ankle. The patient complains of severe right ankle pain when standing. In addition to surgical repair of a compound fracture that has broken through the skin, which of the following treatment plans is important to consider in this patient? A) Application of a topical antibiotic BID until the wound is healed B) Wound irrigation C) Use cold packs three times per day D) Tetanus vaccine and systemic antibiotics

D) Tetanus vaccine and systemic antibiotics A break in the skin with a compound fracture is an indication for use of a tetanus vaccine (if last dose is more than 5 years ago) and systemic antibiotics.

A fracture on the navicular area of the wrist is usually caused by falling forward and landing on the hands. The affected wrist is hyperextended to break the fall. The nurse practitioner is aware that all of the following statements are true regarding a fracture of the scaphoid bone of the wrist except: A) It has a higher rate of nonunion compared with the other bones in the wrist when it is fractured B) The fracture frequently does not show up on an x-ray film when it is taken immediately after the injury C) The x-ray film will show the fracture if the film is repeated in 2 weeks D) These fractures always require surgical intervention to stabilize the joint

D) These fractures always require surgical intervention to stabilize the joint Common signs and symptoms of a fracture of the navicular area include pain, swelling, and tenderness over the thumb side of the wrist, and "crunchiness" and pain with gripping motions. Immediately following the injury, the fracture may not be found on x-ray, leading to misdiagnosis as a sprain. This fracture may be more accurately diagnosed with a bone scan if it does not appear on an x-ray. However, fracture is usually visible if the x-ray is repeated in 2 weeks. Treatment depends largely on the severity and shape of the fracture line. Fractures that are not displaced (those where the break line is small) are immobilized (casting). Nondisplaced fractures that do not heal after 3 to 4 months often require surgical intervention, and the use of other modalities, such as electrical stimulation.

Acanthosis nigricans is associated with all of the following disorders except: A) Obesity B) Diabetes C) Colon cancer D) Tinea versicolor

D) Tinea versicolor Acanthosis nigricans is a benign skin condition that is a sign of insulin resistance. It appears as hyperpigmented velvety areas that are usually located on the neck and the axillae. It is rarely associated with some types of adenocarcinoma of the gastrointestinal tract. Tinea versicolor is a superficial infection of the skin (stratum corneum layer) that is caused by dermatophytes (fungi) of the tinea family. Another name for it is sunspots.

A 17-year-old boy reports feeling something on his left scrotum. On palpation, soft and movable blood vessels that feel like a "bag of worms" are noted underneath the scrotal skin. The testicle is not swollen or reddened. The most likely diagnosis is: A) Chronic orchitis B) Chronic epididymitis C) Testicular torsion D) Varicocele

D) Varicocele Palpation of varicose veins, described as a "bag of worms," in the scrotum is a classic symptom of a varicocele, an abnormal tortuosity and dilation of the veins of the pampiniform plexus within the spermatic cord. It is most common on the left side and may be associated with pain. It occurs in boys and young men and is associated with reduced fertility. The condition is often visible only when the patient is standing. Chronic epididymitis and chronic orchitis are caused by a bacterial infection and commonly cause burning, urinary frequency, and pain. Testicular torsion is an emergent condition in which the testicle becomes twisted, interrupting the blood supply to the testis; to avoid damage, the condition must be corrected within 6 hours.

A photograph of a skin lesion is included in this question. (This image can be seen on the flip side of this card) What is the patient's diagnosis? A) Basal cell carcinoma B) Squamous cell carcinoma C) Nevus D) Melanoma

What is the patient's diagnosis? A) Basal cell carcinoma B) Squamous cell carcinoma C) Nevus D) Melanoma D) Melanoma This is a photograph of melanoma. Think of the "ABCD" rule for pigmented lesions that are highly suspicious: A (asymmetry), B (irregular borders), C (three or more colors such as black, blue, tan, red, white), and D (diameter >6 mm or >¼ inch).

A photograph of a skin lesion is included in this question. (This image can be seen on the flip side of this card) What is the patient's diagnosis? A) Nevus B) Melanoma C) Basal cell carcinoma D) Squamous cell carcinoma

What is the patient's diagnosis? A) Nevus B) Melanoma C) Basal cell carcinoma D) Squamous cell carcinoma C) Basal cell carcinoma This is a photograph of basal cell carcinoma, the most common type of skin cancer. Although the lesion does not have central ulceration, it contains telangiectasia with a waxy or pearly (shiny) appearance. The gold-standard diagnosis for any type of skin cancer is a biopsy of the lesion that is sent to the lab for pathological evaluation.


Conjuntos de estudio relacionados

AP United States History: The Indian Wars

View Set

Chapter 29: Orthopaedic Injuries

View Set

JCAC Module 16, Forensics Methodology & Malware Analysis

View Set

Business Finance Quizzes 1-3 FINC 350

View Set

Chapter 10 Writing Correct and Effective Sentences

View Set

Micro Lecture Chemical and Physical Control

View Set

"First Aid- Chapter 15: Sudden Illnesses"

View Set